You are on page 1of 114

Giordano Company purchased the net assets of Hanes Company on January 1, Year 1, and

made the following entry to record the purchase:

Current assets 100,000


Equipment 150,000
Land 50,000
Buildings 300,000
Goodwill 100,000
Liabilities 80,000
Common 100,000
stock, P1 par
Paid-in 520,000
capital in excess of par
A contingent consideration agreement was made on January 1, Year 1, wherein
additional shares would be issued on January 1, Year 3, to compensate for any fall in
the value of Giordano common stock below P6 per share. The settlement would be to
cure the deficiency by issuing added shares based on their fair value on January 1,
Year 3. The market price of the shares on January 1, Year 3, was P4. How many
shares will Giordano still issue on January 1, Year 3?
50,000 100,000 20,000 51,667
General Feedback
50,000

Mountain Inc. acquired on January 1, Year 2 all the issued and outstanding common
shares of Racer Inc. for 310,000. On this day, the net assets of Racer Inc., amounts to
270,000 including goodwill of 50,000. Per appraisal, plant and equipment and
merchandise inventory were undervalued by 30,000 and overvalued by 15,000,
respectively.

What is the amount of goodwill resulting from this transaction?

25,000

75,000

125,000
General Feedback

Consideration transferred 310,000.00


Net Assets of acquiree - FV
Net Assets - Book value 270,000.00
Acquiree goodwill (50,000.00)
PPE undervaluation 30,000.00

Inventory overvaluation (15,000.00) 235,000.00

Goodwill 75,000.00

Patrick Company acquired the assets (except for cash) and assumed the liabilities of
Steve Company on January 2, Year 1 and Steve Company is dissolved. As
compensation, Patrick Company gave 24,000 shares of its common stock, 12,000
shares of its 8% preferred stock, and cash of 240,000 to the stockholders of Steve
Company. On the date of acquisition, Patrick Company had the following
characteristics:

Common , par value P5; fair value, P20


Preferred, par value P100; fair value, P 100

Immediately prior to acquisition, Steve Company’s balance sheet was as follows:


Cash 132,000 Current Liabilities 228,000
Accounts 170,000 Bonds payable, 400,000
Receivable (net of 10%
P4,000 allowance)
Inventory – LIFO 200,000 Common Stock, P5 600,000
cost Par value
Land 384,000 Additional Paid-in 380,000
Capital
Buildings and 1,032,000 Retained Earnings 310,000
Equipment (net)
Total 1,918,000 Total 1,918,000

An appraisal of Steve Company showed that the fair values of its assets and liabilities
were equal to their book values except for the following, which had fair values as
indicated:
Accounts 158,000 Land 540,000
receivable
Inventory 412,000 Bonds payable 448,000

How much must be the goodwill recognized as a result of this business combination?
322,000 454,000 94,000 0
General Feedback
454,000

On 1 October 2013 BDO Company acquired 100% of PCI Company when the fair
value of PCI’s net assets was P116 million and their carrying amount was P120
million. The consideration transferred comprised P200 million in cash transferred at
the acquisition date, plus another P60 million in cash to be transferred 11 months after
the acquisition date if a specified profit target was met by PCI. At the acquisition
date there was only a low probability of the profit target being met, so the fair value
of the additional consideration liability was P10 million. In the event, the profit target
was met and the P60 million cash was transferred. What amount should BDO present
for goodwill in its statement of consolidated financial position at 31 December 2014,
according to IFRS3 Business combinations?
P94 million P 80 million P 84 million P 144 million
General Feedback
P94 million

On April 1, Year 1, Parson Corp. purchased 80% of the outstanding stock of Sloan Corp.
for $700,000 cash. Parson determined that the fair value of the net identifiable assets was
$800,000 on the date of acquisition. The fair value of Sloan’s stock at date of acquisition
was $18 per share. Sloan had a total of 50,000 shares of stock issued and outstanding
prior to the acquisition. What is the amount of goodwill that should be recorded by
Parson at date of acquisition? $0 $ 60,000 $ 80,000 $120,000
General Feedback
The correct answer is calculated as illustrated below. Assets transferred $700,000 Plus:
Noncontrolling interest in Sloan 10,000 shares × $18 180,000 Less: Fair value of net
identifiable assets of Sloan (800,000)
Goodwill recognized $ 80,00

On January 1, Year 1 the fair value of Pink Conrad , net assets were as follows:
Current assets 100,000
Equipment 150,000
Land 50,000
Buildings 300,000
Liabilities 80,000

On January 1, Year 1 Blue George Company purchased the net assets of the pink
Conrad Company by issuing 100,000 shares of its 1 par value stock when the fair
value of the stock was 6.20 . It was further agreed that Blue George would pay an
additional amount on January 1, 2013 , if the average income during the 2 year period
of Year 1 – 2012 exceeded 80,000 per year . The expected value of this consideration
was calculated as 184,000, the measurement period is one year.
What amount will be recorded as goodwill on January 1, Year 1?
Zero 100,000 180,000 284,000
General Feedback
Pink Conrad
Consideration Transferred
Shares (100,000shares @ 6.20) 620,000.00
Contingent Consideration 184,000.00
Non-Controlling Interest -
Previously-Held Shares -
(520,000.00
Net Assets - Acquiree )
Goodwill 284,000.00

Giordano Company purchased the net assets of Hanes Company on January 1, Year 1, and
made the following entry to record the purchase:

Current assets 100,000


Equipment 150,000
Land 50,000
Buildings 300,000
Goodwill 100,000
Liabilities 80,000
Common stock, P1 par 100,000
Paid-in capital in excess of 520,000
par
A contingent consideration agreement was made on Jan. 1, Year 1, wherein an
additional cash payment would be made on Jan. 1, Year 3, equal to twice the amount
by which average annual earnings of the Hanes Division exceed P25,000 per year,
prior to January 1, Year 3. Net income was P50,000 in Year 1 and P60,000 in Year 2.
How much adjustment will be made to goodwill on January 1, Year 3?
P60,000 P120,000 P85,000 none
General Feedback
None

100% of the equity share capital of Richway Company was acquired by Sunlife
Company on 30 June Year 2. Sunlife issued 500,000 new P1 ordinary shares which
had a fair value of 8 each at the acquisition date. In addition the acquisition resulted
in Sunlife incurring fees payable to external advisers of 200,000 and share issue costs
of 180,000.

In accordance with IFRS3 Business combinations , goodwill at the acquisition date is


measured by subtracting the identifiable assets acquired and the liabilities assumed
from

4,000,000

4,180,000

4,200,000
4,200,000
General Feedback
Answer (d) is CORRECT - In a business combination accounted for as an acquisition, the
fair market value of the net assets is used as the valuation basis for the combination. In
this case, the net assets of the subsidiary have an implied fair market value of $3,600,000
which is the value of the common stock issued to Saxe’s shareholders (200,000 × $18).
Since $3,600,000 is the basis for recording this purchase, the common stock issued is
recorded at $2,000,000 (200,000 shares × $10 par value per share) and additional paid-in
capital is recorded at $1,600,000 ($3,600,000 – $2,000,000). Therefore, answer (d) is
correct because additional paid-in capital should be reported at $2,900,000 ($1,300,000 +
$1,600,000).

On January 1, Year 1, Gulliver Corporation acquired 80 percent of Sea-Gull Company's


common stock for 160,000 cash. The fair value of the non-controlling interest at that date
was determined to be 40,000. Data from the balance sheets of the two companies included
the following amounts as of the date of acquisition:
Jonathan Sea-Gull
Corporation Corporation
Cash 60,000 20,000
Accounts Receivable 80,000 30,000
Inventory 90,000 40,000
Land 100,000 40,000
Buildings and Equipment 200,000 150,000
Less: Accumulated Depreciation (80,000) (50,000)
Investment in Sea-Gull Corporation 160,000
Stock
Total Assets 610,000 230,000

Accounts Payable 100,000 30,000


Bonds Payable 95,000 40,000
Common Stock 200,000 40,000
Retained Earnings 205,000 120,000
Total Liabilities and Stockholders’ 600,000 230,000
Equity

At the date of the business combination, the book values of Sea-Gull's net assets and
liabilities approximated fair value except for inventory, which had a fair value of 45,000,
and land, which had a fair value of 60,000.

Based on the preceding information, what amount of goodwill will be reported in the
consolidated balance sheet prepared immediately after the business combination?

0 40,000 20,000 15,000


General Feedback
Gulliver Corporation
Consideration Transferred 160,000.00
Non-Controlling Interest 40,000.00
Previously-Held Shares -
Net Assets (185,000.00)
Goodwill 15,000.00

On December 31, Year 1, Saxe Corporation was acquired by Poe Corporation. In the
business combination, Poe issued 200,000 shares of its $10 par common stock, with a
market price of $18 a share, for all of Saxe’s common stock. The stockholders’ equity
section of each company’s balance sheet immediately before the combination was
Poe Saxe
Common stock $3,000,000 $1,500,000
Additional paid-in capital 1,300,000 150,000
Retained earnings 2,500,000 850,000
$6,800,000 $2,500,000

In the December 31, Year 1 consolidated balance sheet, additional paid-in capital should
be reported at $ 950,000 $1,300,000 $1,450,000 $2,900,000
General Feedback
In a business combination accounted for as an acquisition, the fair market value of the net
assets is used as the valuation basis for the combination. In this case, the net assets of the
subsidiary have an implied fair market value of $3,600,000 which is the value of the
common stock issued to Saxe’s shareholders (200,000 × $18). Since $3,600,000 is the
basis for recording this purchase, the common stock issued is recorded at $2,000,000
(200,000 shares × $10 par value per share) and additional paid-in capital is recorded at
$1,600,000 ($3,600,000 – $2,000,000). Therefore, answer (d) is correct because
additional paid-in capital should be reported at $2,900,000 ($1,300,000 + $1,600,000).

On December 31, Year 1, Neal Co. issued 100,000 shares of its $10 par value common
stock in exchange for all of Frey Inc.’s outstanding stock. The fair value of Neal’s
common stock on December 31, Year 1, was $19 per share. The carrying amounts and
fair values of Frey’s assets and liabilities on December 31, Year 1, were as follows:

Carrying amount Fair value


Cash 240,000 240,000
Receivables 270,000 270,000
Inventory 435,,000 405,000
Property, plant, and equipment 1,305,000 1,440,000
Liabilities (525,000) (525,000)
Net assets 1,725,000 1,830,000
What is the amount of goodwill resulting from the business combination?
$175,000 $105,000 $ 70,000 $0
General Feedback
(b) The direct costs of acquisition should be an expense of the period in a business
combination accounted for by the acquisition method. General expenses related to the
acquisition are also deducted as incurred in determining the combined corporation’s net
income for the current period.
70,000

On 1 January Year 1, Vinculum acquired 75% of Pathways’ equity shares by


means of an immediate share exchange of two shares in Vinculum for five shares in
Pathways. The fair value of Vinculum and Pathways’ shares on 1 January Year 1 were
4.00 and 3.00 respectively. In addition to the share exchange, Vinculum will make a
cash payment of 1.32 per acquired share, deferred until 1 January Year 2. Vinculum
has not recorded any of the consideration for Pathways in its financial statements.
Vinculum’s cost of capital is 10% per annum.
The summarized statements of financial position of the two companies as at 30 June
Year 1 are:
Vinculum Pathways
Assets
Non-current assets (note (ii))
Property, plant and equipment 55,000 28,600
Financial asset equity investments (note 11,500 6,000
(v))
66,500 34,600
Current assets
Inventory (note (iv)) 17,000 15,400
Trade receivables (note (iv)) 14,300 10,500
Bank 2,200 1,600
33,500 27,500
Total assets 100,000 62,100
Equity and liabilities
Equity
Equity shares of P1 each 20,000 20,000
Other component of equity 4,000 nil
Retained earnings – at 1 July 2014 26,200 14,000
Retained earnings – for year ended 30 24,000 10,000
June Year 1
74,200 44,000
Current liabilities (note (iv)) 25,800 18,100
Total equity and liabilities 100,000 62,100

The following information is relevant:


(i.) Pathways’ business is seasonal and 60% of its annual profit is made in the period 1
January to 30 June each year.
(ii.) At the date of acquisition, the fair value of Pathways’ net assets was equal to their
carrying amounts with the following exceptions:
a. An item of plant had a fair value of P2 million below its carrying value.
At the date of acquisition it had a remaining life of two years.
b. The fair value of Pathways’ investments was P7 million (see also note
(v)).
c. Pathways owned the rights to a popular mobile (cell) phone game. At
the date of acquisition, a specialist valuer estimated that the rights were worth
P12 million and had an estimated remaining life of five years.
(iii.) Following an impairment review, consolidated goodwill is to be written down by P3
million as at 30 June Year 1.
(iv.) (iv) Vinculum sells goods to Pathways at cost plus 30%. Pathways had P1·8 million
of goods in its inventory at
30 June Year 1 which had been supplied by Vinculum. In addition, on 28 June Year 1,
Vinculum processed the sale of P800,000 of goods to Pathways, which Pathways did not
account for until their receipt on 2 July Year 1. The in-transit reconciliation should be
achieved by assuming the transaction had been recorded in the books of Pathways before
the year end. At 30 June Year 1, Vinculum had a trade receivable balance of P2·4 million
due from Pathways which differed to the equivalent balance in Pathways’ books due to
the sale made on 28 June Year 1.
(v.) At 30 June Year 1, the fair values of the financial asset equity investments of
Vinculum and Pathways were P13·2 million and P7·9 million respectively.
Vinculum’s policy is to value the non-controlling interest at fair value at the date of
acquisition. For this purpose, Pathways’ share price at that date is representative of the
fair value of the shares held by the non-controlling interest.

Determine the Goodwill / (Gain) recognized at acquisition

19,000 8,000 5,000 (10,000)


General Feedback
Controlling interest
Share exchange (20,000 x 75% x 2/5) = (6,000 x $4·00) 24,000
Deferred consideration (20,000 x 75% x $1·32/1·1) 18,000
Non-controlling interest (20,000 x 25% x $3·00) 15,000
–––––––
57,000
Equity shares 20,000
Pre-acquisition retained earnings:
at 30 June 2014 14,000
from 1 July to 31 December 2014 (10,000 x 40%) 4,000
Fair value adjustments – plant (2,000)
– game rights 12,000
– investments 1,000 (49,000)
––––––– –––––––
Goodwill on acquisition 8,000

A parent entity is acquiring a majority holding in an entity whose shares are dealt in on a
recognised market. Under IFRS3 Business combinations , which of the following
measurement bases may be used in measuring the non-controlling interest at the
acquisition date?

The nominal value of the shares in the acquiree not acquired

The fair value of the shares in the acquiree not acquired

The non-controlling interest in the acquiree's assets and liabilities at book value

The non-controlling interest in the acquiree's assets at fair value


General Feedback
The fair value of the shares in the acquiree not acquired

On January 1, Year 1 the fair value of Pink Conrad , net assets were as follows:
Current assets 100,000
Equipment 150,000
Land 50,000
Buildings 300,000
Liabilities 80,000

On January 1, Year 1 Blue George Company purchased the net assets of the pink
Conrad Company by issuing 100,000 shares of its 1 par value stock when the fair
value of the stock was 6.20 . It was further agreed that Blue George would pay an
additional amount on January 1, 2013 , if the average income during the 2 year period
of Year 1 – 2012 exceeded 80,000 per year . The expected value of this consideration
was calculated as 184,000, the measurement period is one year.
What amount will be recorded as goodwill on January 1, Year 1?
Zero 100,000 180,000 284,000
General Feedback
Pink Conrad
Consideration Transferred
Shares (100,000shares @ 6.20) 620,000.00
Contingent Consideration 184,000.00
Non-Controlling Interest -
Previously-Held Shares -
(520,000.00
Net Assets - Acquiree )
Goodwill 284,000.00
In a business combination, an acquirer's interest in the fair value of the net assets
acquired exceeds the consideration transferred in the combination. Under
IFRS3 Business combinations, the acquirer should (select one answer)

recognise the excess immediately in profit or loss

recognise the excess immediately in other comprehensive income

reassess the recognition and measurement of the net assets acquired and the
consideration transferred, then recognise any excess immediately in profit or loss

reassess the recognition and measurement of the net assets acquired and the
consideration transferred, then recognise any excess immediately in other
comprehensive income
General Feedback
reassess the recognition and measurement of the net assets acquired and the
consideration transferred, then recognise any excess immediately in profit or loss

On 1 January Year 1, Vinculum acquired 75% of Pathways’ equity shares by


means of an immediate share exchange of two shares in Vinculum for five shares in
Pathways. The fair value of Vinculum and Pathways’ shares on 1 January Year 1 were
4.00 and 3.00 respectively. In addition to the share exchange, Vinculum will make a
cash payment of 1.32 per acquired share, deferred until 1 January Year 2. Vinculum
has not recorded any of the consideration for Pathways in its financial statements.
Vinculum’s cost of capital is 10% per annum.
The summarized statements of financial position of the two companies as at 30 June
Year 1 are:
Vinculum Pathways
Assets
Non-current assets (note (ii))
Property, plant and equipment 55,000 28,600
Financial asset equity investments (note 11,500 6,000
(v))
66,500 34,600
Current assets
Inventory (note (iv)) 17,000 15,400
Trade receivables (note (iv)) 14,300 10,500
Bank 2,200 1,600
33,500 27,500
Total assets 100,000 62,100
Equity and liabilities
Equity
Equity shares of P1 each 20,000 20,000
Other component of equity 4,000 nil
Retained earnings – at 1 July 2014 26,200 14,000
Retained earnings – for year ended 30 24,000 10,000
June Year 1
74,200 44,000
Current liabilities (note (iv)) 25,800 18,100
Total equity and liabilities 100,000 62,100

The following information is relevant:


(i.) Pathways’ business is seasonal and 60% of its annual profit is made in the period 1
January to 30 June each year.
(ii.) At the date of acquisition, the fair value of Pathways’ net assets was equal to their
carrying amounts with the following exceptions:
a. An item of plant had a fair value of P2 million below its carrying value.
At the date of acquisition it had a remaining life of two years.
b. The fair value of Pathways’ investments was P7 million (see also note
(v)).
c. Pathways owned the rights to a popular mobile (cell) phone game. At
the date of acquisition, a specialist valuer estimated that the rights were worth
P12 million and had an estimated remaining life of five years.
(iii.) Following an impairment review, consolidated goodwill is to be written down by P3
million as at 30 June Year 1.
(iv.) (iv) Vinculum sells goods to Pathways at cost plus 30%. Pathways had P1·8 million
of goods in its inventory at
30 June Year 1 which had been supplied by Vinculum. In addition, on 28 June Year 1,
Vinculum processed the sale of P800,000 of goods to Pathways, which Pathways did not
account for until their receipt on 2 July Year 1. The in-transit reconciliation should be
achieved by assuming the transaction had been recorded in the books of Pathways before
the year end. At 30 June Year 1, Vinculum had a trade receivable balance of P2·4 million
due from Pathways which differed to the equivalent balance in Pathways’ books due to
the sale made on 28 June Year 1.
(v.) At 30 June Year 1, the fair values of the financial asset equity investments of
Vinculum and Pathways were P13·2 million and P7·9 million respectively.
(vi.) Vinculum’s policy is to value the non-controlling interest at fair value at the date of
acquisition. For this purpose, Pathways’ share price at that date is representative of the
fair value of the shares held by the non-controlling interest.

How much is the consolidated Retained Earnings at December 31, Year 1?

53,700 53,400 52,800 50,200


General Feedback
Consolidated retained earnings:
$’000
Palistar’s retained earnings (26,200 + 24,000) 50,200
Stretcher’s adjusted post-acquisition profit (3,200 x 75% see below) 2,400
Finance cost on deferred consideration (18,000 x 10% x 6/12) (900)
URP in inventory (w (iii)) (600)
Gain on equity investments (13,200 – 11,500) 1,700
–––––––
52,800
–––––––
The adjusted post-acquisition profit of Stretcher is:
$10 million x 60% 6,000
Gain on investments (7,900 – 7,000) 900
Reduced depreciation of plant (2,000 x 6/24) 500
Amortisation of game rights (12,000/5 years x 6/12) (1,200)
Goodwill impairment (w (i)) (3,000)
–––––––
3,200

On January 1, Year 1, Gulliver Corporation acquired 80 percent of Sea-Gull Company's


common stock for 160,000 cash. The fair value of the non-controlling interest at that date
was determined to be 40,000. Data from the balance sheets of the two companies included
the following amounts as of the date of acquisition:
Jonathan Sea-Gull
Corporation Corporation
Cash 60,000 20,000
Accounts Receivable 80,000 30,000
Inventory 90,000 40,000
Land 100,000 40,000
Buildings and Equipment 200,000 150,000
Less: Accumulated Depreciation (80,000) (50,000)
Investment in Sea-Gull Corporation 160,000
Stock
Total Assets 610,000 230,000

Accounts Payable 100,000 30,000


Bonds Payable 95,000 40,000
Common Stock 200,000 40,000
Retained Earnings 205,000 120,000
Total Liabilities and Stockholders’ 600,000 230,000
Equity

At the date of the business combination, the book values of Sea-Gull's net assets and
liabilities approximated fair value except for inventory, which had a fair value of 45,000,
and land, which had a fair value of 60,000.

Based on the preceding information, what amount of goodwill will be reported in the
consolidated balance sheet prepared immediately after the business combination?
0 40,000 20,000 15,000
General Feedback
Gulliver Corporation
Consideration Transferred 160,000.00
Non-Controlling Interest 40,000.00
Previously-Held Shares -
Net Assets (185,000.00)
Goodwill 15,000.00

Mountain Inc. acquired on January 1, Year 2 all the issued and outstanding common
shares of Racer Inc. for 310,000. On this day, the net assets of Racer Inc., amounts to
270,000 including goodwill of 50,000. Per appraisal, plant and equipment and
merchandise inventory were undervalued by 30,000 and overvalued by 15,000,
respectively.

What is the amount of goodwill resulting from this transaction?

25,000

75,000

125,000
General Feedback

Consideration transferred 310,000.00


Net Assets of acquiree - FV
Net Assets - Book value 270,000.00
Acquiree goodwill (50,000.00)
PPE undervaluation 30,000.00

Inventory overvaluation (15,000.00) 235,000.00

Goodwill 75,000.00

The excess of the consideration transferred plus the amount of any non-controlling
interest in the acquiree over the identifiable assets and liabilities recognized is
goodwill

Gain from acquisition


minority interest

cost of acquisition
General Feedback
Goodwill

On January 1, Year 1, Gulliver Corporation acquired 80 percent of Sea-Gull Company's


common stock for 160,000 cash. The fair value of the non-controlling interest at that date
was determined to be 40,000. Data from the balance sheets of the two companies included
the following amounts as of the date of acquisition:
Jonathan Sea-Gull
Corporation Corporation
Cash 60,000 20,000
Accounts Receivable 80,000 30,000
Inventory 90,000 40,000
Land 100,000 40,000
Buildings and Equipment 200,000 150,000
Less: Accumulated Depreciation (80,000) (50,000)
Investment in Sea-Gull Corporation 160,000
Stock
Total Assets 610,000 230,000

Accounts Payable 100,000 30,000


Bonds Payable 95,000 40,000
Common Stock 200,000 40,000
Retained Earnings 205,000 120,000
Total Liabilities and Stockholders’ 600,000 230,000
Equity

At the date of the business combination, the book values of Sea-Gull's net assets and
liabilities approximated fair value except for inventory, which had a fair value of 45,000,
and land, which had a fair value of 60,000.
Based on the preceding information, what amount will be reported as total stockholders'
equity in the consolidated balance sheet prepared immediately after the business
combination?

445,000 205,000 565,000 550,000


General Feedback
Common Stock 200,000.00
Retained Earnings 205,000.00
Non-Controlling Interest 40,000.00
Total Shareholders' Equity -
Consolidated 445,000.00
Should the following costs be included in the consideration transferred in a business
combination, according to IFRS3 Business combinations?
(1) Costs of maintaining an acquisitions department.
(2) Fees paid to accountants to effect the combination.
Cost Cost (2)
(1)

No No

No Yes

Yes No

Yes No
General Feedback
No No

AIG Company acquired a 70% interest in EASTWEST Company for 1,960,000 when
the fair value of EASTWEST’s identifiable assets and liabilities was 700,000 and
elected to measure the non-controlling interest at its share of the identifiable net
assets. Annual impairment reviews of goodwill have not resulted in any impairment
losses being recognized.

EASTWEST's current statement of financial position shows share capital of 100,000,


a revaluation reserve of 300,000 and retained earnings of 1,400,000.

Under IFRS3 Business combinations, what figure in respect of goodwill should now
be carried in AIG’s consolidated statement of financial position?
1,470,000 160,000 1,260,000 700,000
General Feedback
1,470,000

On January 1, Year 1, Polk Corp. and Strass Corp. had condensed balance sheets as
follows:
Polk Strass
Current assets $ 70,000 $20,000
Noncurrent assets 90,000 40,000
Total assets $160,000 $60,000
Current liabilities 30,000 10,000
Long-term debt 50,000 --
Stockholders’ equity 80,000 50,000
Total liabilities and stockholders’ equity 160,000 60,000
On January 2, Year 1, Polk borrowed $60,000 and used the proceeds to purchase 90% of
the outstanding common shares of Strass. This debt is payable in ten equal annual
principal payments, plus interest, beginning December 30, Year 1. The excess cost of the
investment over Strass’ book value of acquired net assets should be allocated 60% to
inventory and 40% to goodwill. On January 1, Year 1, the fair
value of Polk shares held by noncontrolling parties was $10,000.

On Polk’s January 2, Year 1 consolidated balance sheet


Stockholders’ equity including noncontrolling interests should be

$ 80,000 $ 85,000 $ 90,000 $130,000


General Feedback
In the consolidated balance sheet, neither the parent company’s investment account nor
the subsidiary’s stockholders’ equity is reported. These amounts are eliminated in the
same journal entry that records the excess of cost over book value. The portion of the
subsidiary’s stockholders’ equity that is noteliminated is reported as noncontrolling
interest in the equity section of the consolidated balance sheet. Therefore, the parent’s
stockholders’ equity ($90,000) equals the consolidated stockholders’ equity plus the
minority interest. Note that once the candidate has completed items 16 through 20, the
answers can be checked using the balance sheet equation.

Current
assets
+
Noncurrent
assets
=
Current
liabilities
+
Noncurrent
liabilities
+
Stockholders’
equity
$102,000 + $138,000 = $46,000 + $104,000 + $90,000

A subsidiary, acquired for cash in a business combination, owned inventories with a


market value greater than the book value as of the date of combination. A consolidated
balance sheet prepared immediately after the acquisition would include this difference as
part of Deferred credits Goodwill Inventories Retained earnings
General Feedback
The assets acquired would be revalued to their fair market value. Answer (c) is correct
because the inventory account would then include the difference between the market
value and book value. Answer (a) is incorrect because a deferred credit is never recorded.
Answer (b) is incorrect because goodwill represents the excess of cost plus the fair value
of previously held interests plus the fair value of non-controlling interest less the fair
value of net identifiable assets. Answer (d) is incorrect because the retained earnings
account is not affected by this transaction when acquisition accounting is used.

Which of the following expenses related to the business acquisition should be


included, in total, in the determination of net income of the combined corporation for
the period in which the expenses are incurred?
Fees of finders Registration fees for
and consultants equity securities
issued

Yes Yes

Yes No

No Yes

No No
General Feedback
Acquisition costs such as finder’s fees are expensed in the period incurred. Registration
fees for equity securities are a reduction in the issue price of the securities. Therefore,
answer (b) is correct.

On 1 January Year 1, Vinculum acquired 75% of Pathways’ equity shares by


means of an immediate share exchange of two shares in Vinculum for five shares in
Pathways. The fair value of Vinculum and Pathways’ shares on 1 January Year 1 were
4.00 and 3.00 respectively. In addition to the share exchange, Vinculum will make a
cash payment of 1.32 per acquired share, deferred until 1 January Year 2. Vinculum
has not recorded any of the consideration for Pathways in its financial statements.
Vinculum’s cost of capital is 10% per annum.
The summarized statements of financial position of the two companies as at 30 June
Year 1 are:
Vinculum Pathways
Assets
Non-current assets (note (ii))
Property, plant and equipment 55,000 28,600
Financial asset equity investments (note 11,500 6,000
(v))
66,500 34,600
Current assets
Inventory (note (iv)) 17,000 15,400
Trade receivables (note (iv)) 14,300 10,500
Bank 2,200 1,600
33,500 27,500
Total assets 100,000 62,100
Equity and liabilities
Equity
Equity shares of P1 each 20,000 20,000
Other component of equity 4,000 nil
Retained earnings – at 1 July 2014 26,200 14,000
Retained earnings – for year ended 30 24,000 10,000
June Year 1
74,200 44,000
Current liabilities (note (iv)) 25,800 18,100
Total equity and liabilities 100,000 62,100

The following information is relevant:


(i.) Pathways’ business is seasonal and 60% of its annual profit is made in the period 1
January to 30 June each year.
(ii.) At the date of acquisition, the fair value of Pathways’ net assets was equal to their
carrying amounts with the following exceptions:
a. An item of plant had a fair value of P2 million below its carrying value.
At the date of acquisition it had a remaining life of two years.
b. The fair value of Pathways’ investments was P7 million (see also note
(v)).
c. Pathways owned the rights to a popular mobile (cell) phone game. At
the date of acquisition, a specialist valuer estimated that the rights were worth
P12 million and had an estimated remaining life of five years.
(iii.) Following an impairment review, consolidated goodwill is to be written down by P3
million as at 30 June Year 1.
(iv.) (iv) Vinculum sells goods to Pathways at cost plus 30%. Pathways had P1·8 million
of goods in its inventory at
30 June Year 1 which had been supplied by Vinculum. In addition, on 28 June Year 1,
Vinculum processed the sale of P800,000 of goods to Pathways, which Pathways did not
account for until their receipt on 2 July Year 1. The in-transit reconciliation should be
achieved by assuming the transaction had been recorded in the books of Pathways before
the year end. At 30 June Year 1, Vinculum had a trade receivable balance of P2·4 million
due from Pathways which differed to the equivalent balance in Pathways’ books due to
the sale made on 28 June Year 1.
(v.) At 30 June Year 1, the fair values of the financial asset equity investments of
Vinculum and Pathways were P13·2 million and P7·9 million respectively.
(vi.) Vinculum’s policy is to value the non-controlling interest at fair value at the date of
acquisition. For this purpose, Pathways’ share price at that date is representative of the
fair value of the shares held by the non-controlling interest.

How much is the Non-Controlling Interest to be presented in the December 31, Year 1
consolidated statement of financial position?

17,500 16,550 15,800 15,000


General Feedback
Non-controlling interest
Fair value on acquisition (w (i)) 15,000
Post-acquisition profit (3,200 x 25% (w (iii)) 800
–––––––
15,800

100% of the equity share capital of Richway Company was acquired by Sunlife
Company on 30 June Year 2. Sunlife issued 500,000 new P1 ordinary shares which
had a fair value of 8 each at the acquisition date. In addition the acquisition resulted
in Sunlife incurring fees payable to external advisers of 200,000 and share issue costs
of 180,000.

In accordance with IFRS3 Business combinations , goodwill at the acquisition date is


measured by subtracting the identifiable assets acquired and the liabilities assumed
from

4,000,000

4,180,000

4,200,000

4,200,000
General Feedback
4,000,000

Mountain Inc. acquired on January 1, Year 2 all the issued and outstanding common
shares of Racer Inc. for 310,000. On this day, the net assets of Racer Inc., amounts to
270,000 including goodwill of 50,000. Per appraisal, plant and equipment and
merchandise inventory were undervalued by 30,000 and overvalued by 15,000,
respectively.

What is the amount of goodwill resulting from this transaction?

25,000

75,000

125,000
General Feedback
Consideration transferred
310,000.00
Net Assets of acquiree - FV
Net Assets - Book value 270,000.00
Acquiree goodwill (50,000.00)
PPE undervaluation 30,000.00

Inventory overvaluation (15,000.00) 235,000.00

Goodwill 75,000.00

On December 31, Year 1, Saxe Corporation was acquired by Poe Corporation. In the
business combination, Poe issued 200,000 shares of its $10 par common stock, with a
market price of $18 a share, for all of Saxe’s common stock. The stockholders’ equity
section of each company’s balance sheet immediately before the combination was
Poe Saxe
Common stock $3,000,000 $1,500,000
Additional paid-in capital 1,300,000 150,000
Retained earnings 2,500,000 850,000
$6,800,000 $2,500,000

In the December 31, Year 1 consolidated balance sheet, common stock should be reported
at $3,000,000 $3,500,000 $4,000,000 $5,000,000
General Feedback
In a business combination, the common stock account of the combined entity is the
number of shares outstanding multiplied by the par value of the stock. The total common
stock account of the combined entity is equal to $5,000,000, the $3,000,000 originally
outstanding plus the total par value of the stock issued in the acquisition, $2,000,000
(200,000 × $10).

On January 1, Year 1, Lake Corporation acquired 100% of the outstanding common


stock of Shore Corporation for $800,000. On the date of acquisition, the fair value of
Shore’s net identifiable assets is $820,000. The book value of Shore Corporation’s net
assets is $760,000. In Lake’s Year 1 financial statements, Lake should recognize
Goodwill on the balance sheet A gain from bargain purchase A reduction in
certain noncurrent assets on the balance sheet
An extraordinary gain.
General Feedback
A gain from bargain purchase
is incorrect - goodwill is only recognized when the purchase price is greater than the fair
value of the recorded assetsis CORRECT - The consideration paid plus the fair value of
the noncontrolling interest plus the fair value of any previous purchases of common stock
is less than the fair value of the net identifiable assets acquired, the acquirer should
recognize a gain from bargain purchase on the income statement. The gain would be
equal to $20,000 ($820,000 – 800,000).is incorrect - it describes an inappropriate
methodis incorrect - the gain is not treated as extraordinary

On October 1, Year 1, Water Corporation acquired all the assets and assumed all the
liabilities of Gulaman Company by issuing 20,000 shares with a fair value of P67.5 per
share and an obligation to pay a contingent consideration with a fair value of P750,000.
In addition, Water paid the following acquisition related costs:

Legal fees P105,600


Audit fee for SEC registration of 320,400
stock issue
Costs of stock certificates 35,000
Broker’s fee 49,000
Other direct cost of acquisition 50,000
General and allocated expenses 14,000

The Statement of Financial Position as of September 30, Year 1 of Water and Gulaman,
together with the fair market value of the assets and liabilities are presented below:

Water Gulaman
Book Value Fair Value Book Value Fair Value
Cash P640,000 P640,000 P45,000 P45,000
Accounts 360,000 335,000 70,000 54,000
receivable
Inventories 475,000 390,000 87,000 78,000
Prepaid 25,000 - 13,500 5,000
expenses
Land 2,000,000 2,900,000 900,000 1,550,000
Building 800,000 900,000 723,000 768,000
Equipment 700,000 585,000 361,500 360,000
Goodwill - - 300,000 -
Total assets P5,000,000 P5,750,000 P2,500,000 P2,860,000

Accounts 312,500 312,500 200,000 200,000


payable
Notes payable 937,500 980,000 700,000 765,000
Capital stock, 2,000,000 850,000
50 par
Additional 1,000,000 400,000
paid in capital
Retained 750,000 350,000
earnings
Total equities P5,000,000 P2,500,000

Compute for the balances that will be shown on the October 1, Year 1 statement of
financial position of the surviving company:
Retained earnings

480,000 540,000 526,000 475,000


General Feedback
526,000

Are the following statements about an acquisition true or false, according to IFRS3
(2008) Business combinations?
a. The acquirer should recognise the acquiree's contingent liabilities if certain
conditions are met.
a. The acquirer should recognise the acquiree's contingent assets if certain conditions
are met.
Statement Statement
(1) (2)

False False

False True

True False

True True
General Feedback
True False

On August 31, Year 2, Wood Corp. issued 100,000 shares of its $20 par value
common stock for the net assets of Pine, Inc., in a business combination accounted for
by the acquisition method. The market value of Wood’s common stock on August 31
was $36 per share. Wood paid a fee of $160,000 to the consultant who arranged this
acquisition. Costs of registering and issuing the equity securities amounted to
$80,000. No goodwill was involved in the purchase. What amount should Wood
capitalize as the cost of acquiring Pine’s net assets?

$3,600,000

$3,680,000

$3,760,000

$3,840,000
General Feedback
In a business combination accounted for as an acquisition, the fair market value of the net
assets is used as the valuation basis for the combination. In this case, the net assets of the
subsidiary have an implied fair market value of $3,600,000 which is the value of the
common stock issued to Pine’s shareholders (100,000 shares × $36). The direct cost of
acquisition should not be included as part of the cost of a company acquired, and the cost
of registering equity securities should be a reduction of the issue price of the securities
(i.e., additional paid-in capital). Thus, the $160,000 paid for a consultant who arranged
the acquisition should be expensed, and the $80,000 cost for registering and issuing the
equity securities should be treated as a reduction of additional paid-in capital. Answer (a)
is correct because the total amount to be capitalized is $3,600,000.

100% of the equity share capital of Richway Company was acquired by Sunlife
Company on 30 June Year 2. Sunlife issued 500,000 new P1 ordinary shares which
had a fair value of 8 each at the acquisition date. In addition the acquisition resulted
in Sunlife incurring fees payable to external advisers of 200,000 and share issue costs
of 180,000.

In accordance with IFRS3 Business combinations , goodwill at the acquisition date is


measured by subtracting the identifiable assets acquired and the liabilities assumed
from

4,000,000

4,180,000

4,200,000

4,200,000
General Feedback
4,000,000

The Grand Company will issue share at 10 par value common stock for all the net
assets of the Nuts Company. Grand’s common has current market value of 40 per
share. Nuts balance sheet accounts are shown below:
Current assets 320,000
Property and equipment 880,000
Liabilities 400,000
Common stock, P4 par 80,000
Additional paid-in capital 320,000
Retained earnings 400,000
The fair value of current assets is 400,000 while that of property and equipment is
1,600,000. All the liabilities are correctly stated. Grand issued sufficient shares so that
the fair market value of the stock equals the fair market value of Nuts net assets plus
goodwill of 200,000. How much must be the cost of investment if goodwill of 200,000
must be recognized?
2,200,000

1,800,000

1,600,000

200,000
General Feedback
1,800,000

Mountain Inc. acquired on January 1, Year 2 all the issued and outstanding common
shares of Racer Inc. for 310,000. On this day, the net assets of Racer Inc., amounts to
270,000 including goodwill of 50,000. Per appraisal, plant and equipment and
merchandise inventory were undervalued by 30,000 and overvalued by 15,000,
respectively.

What is the amount of goodwill resulting from this transaction?

25,000

75,000

125,000
General Feedback

Consideration transferred 310,000.00


Net Assets of acquiree - FV
Net Assets - Book value 270,000.00
Acquiree goodwill (50,000.00)
PPE undervaluation 30,000.00

Inventory overvaluation (15,000.00) 235,000.00

Goodwill 75,000.00

Companies A and B decide to consolidate , Asset and estimated annual earnings


contribution are as follows:
Co. A Co. B Total
Net asset contribution 300,000 400,000 700,000
Estimated annual earnings
Contribution 50,000 80,000 130,000

Stock holder of the two companies agreed that a single class of stock be issued that
their contribution be measured by net assets plus allowances for a goodwill, and that
10% considered as a normal rate of return ,Earnings in excess of the normal rate of
return sales be capitalized at 20% in the calculating goodwill , it was also agreed that
the authorized capital stock of the new corporation shall be 20,000 shares with a par
value of 100 a shares.

Determine: (1) The amount of goodwill credited to Co. A; and (2) the total
contribution of Co. B

(1) 100,000; (2) 400,000

1) 150,000; (2) 500,000

1)100,000; (2) 600,000

(1) 200,000; (2) 600,000


General Feedback
Note: this is an old structure
problem
Companies A and B A Co. B Co.
Net Asset Contribution 300,000.00 400,000.00
Multiply by: Normal Rate of Return 0.10 0.10
Normal Rate of Return 30,000.00 40,000.00
Estimated Annual Earnings 50,000.00 80,000.00
Estimated Excess Earnings 20,000.00 40,000.00
Divide by: Capitalization rate 0.20 0.20
Goodwill 100,000.00 200,000.00
Net Asset Contribution 300,000.00 400,000.00
Total Contribution 400,000.00 600,000.00

AIG Company acquired a 70% interest in EASTWEST Company for 1,960,000 when
the fair value of EASTWEST’s identifiable assets and liabilities was 700,000 and
elected to measure the non-controlling interest at its share of the identifiable net
assets. Annual impairment reviews of goodwill have not resulted in any impairment
losses being recognized.

EASTWEST's current statement of financial position shows share capital of 100,000,


a revaluation reserve of 300,000 and retained earnings of 1,400,000.

Under IFRS3 Business combinations, what figure in respect of goodwill should now
be carried in AIG’s consolidated statement of financial position?
1,470,000 160,000 1,260,000 700,000
General Feedback
1,470,000

On October 1, Year 1, Water [A1] Corporation acquired all the assets and assumed all the
liabilities of Gulaman Company by issuing 20,000 shares with a fair value of P67.5 per
share and an obligation to pay a contingent consideration with a fair value of P750,000.
In addition, Water paid the following acquisition related costs:

Legal fees P105,600


Audit fee for SEC registration of 320,400
stock issue
Costs of stock certificates 35,000
Broker’s fee 49,000
Other direct cost of acquisition 50,000
General and allocated expenses 14,000

The Statement of Financial Position as of September 30, Year 1 of Water and Gulaman,
together with the fair market value of the assets and liabilities are presented below:

Water Gulaman
Book Value Fair Value Book Value Fair Value
Cash P640,000 P640,000 P45,000 P45,000
Accounts 360,000 335,000 70,000 54,000
receivable
Inventories 475,000 390,000 87,000 78,000
Prepaid 25,000 - 13,500 5,000
expenses
Land 2,000,000 2,900,000 900,000 1,550,000
Building 800,000 900,000 723,000 768,000
Equipment 700,000 585,000 361,500 360,000
Goodwill - - 300,000 -
Total assets P5,000,000 P5,750,000 P2,500,000 P2,860,000

Accounts 312,500 312,500 200,000 200,000


payable
Notes payable 937,500 980,000 700,000 765,000
Capital stock, 2,000,000 850,000
50 par
Additional 1,000,000 400,000
paid in capital
Retained 750,000 350,000
earnings
Total equities P5,000,000 P2,500,000
Compute for the balances that will be shown on the October 1, Year 1 statement of
financial position of the surviving company:
Total assets
7,015,000 6,980,000 7,118,000 7,491,000
General Feedback
7,491,000

Mango Inc. acquired on January 1, 2013 all the issued and outstanding common shares
of Celine Inc. for P310,000 and Celine Inc. is dissolved. On this day, the assets and
liabilities of Celine Inc. show:

Cash 30,000
Merchandise inventory 90,000
Plant and equipment 160,000
Goodwill 50,000
Liabilities (60,000)

Per appraisal, plant and equipment and merchandise inventory were valued at 190,000
and 75,000, respectively. What is the amount of goodwill resulting from this
transaction?

125,000

40,000

75,000

90,000
General Feedback
75,000

On January 1, Year 1, Dabawenyo Company purchased 80 percent of the outstanding


shares of Minero Company for 600,000 in cash. On that date, Minero had 200,000 of
capital stock and 500,000 of retained earnings. All of the assets and liabilities of Minero
Company were fairly valued. Goodwill, if any, is not amortized. The January 1, Year 5,
inventory of Minero includes 10,000 of merchandise purchased from Dabawenyo in year
Year 4 at 125 percent of cost. The December 31, Year 5, inventory of Minero includes
16,000 of merchandise purchased from Dabawenyo at the same markup. Minero’s
inventories are on a FIFO basis. For year Year 5, Minero reported net income of 80,000
and paid dividends of 40,000. Dabawenyo’s income from its own operations is 400,000; it
paid dividends of 250,000. If Dabawenyo uses the cost method on its books, what will be
its consolidated net income to retained earnings compared with its book income?
32,000 higher 30,800 higher 29,800 higher 1,200 lower
General Feedback
EHP NCI Conso

Income - Parent (Dabawenyo) 400,000.00 400,000.00

Income - Subsidiary (Minero) 64,000.00 16,000.00 80,000.00


Intercompany sale -
Downstream

RGP from Inventory, beg 2,000.00 2,000.00

DGP from Inventory, end (3,200.00) (3,200.00)


Total
462,800.00 16,000.00 478,800.00

On January 1, Year 1, Gulliver Corporation acquired 80 percent of Sea-Gull Company's


common stock for 160,000 cash. The fair value of the non-controlling interest at that date
was determined to be 40,000. Data from the balance sheets of the two companies included
the following amounts as of the date of acquisition:
Jonathan Sea-Gull
Corporation Corporation
Cash 60,000 20,000
Accounts Receivable 80,000 30,000
Inventory 90,000 40,000
Land 100,000 40,000
Buildings and Equipment 200,000 150,000
Less: Accumulated Depreciation (80,000) (50,000)
Investment in Sea-Gull Corporation 160,000
Stock
Total Assets 610,000 230,000

Accounts Payable 100,000 30,000


Bonds Payable 95,000 40,000
Common Stock 200,000 40,000
Retained Earnings 205,000 120,000
Total Liabilities and Stockholders’ 600,000 230,000
Equity

At the date of the business combination, the book values of Sea-Gull's net assets and
liabilities approximated fair value except for inventory, which had a fair value of 45,000,
and land, which had a fair value of 60,000.

Based on the preceding information, what amount of goodwill will be reported in the
consolidated balance sheet prepared immediately after the business combination?
0 40,000 20,000 15,000
General Feedback
Gulliver Corporation
Consideration Transferred 160,000.00
Non-Controlling Interest 40,000.00
Previously-Held Shares -
Net Assets (185,000.00)
Goodwill 15,000.00
Companies A and B decide to consolidate , Asset and estimated annual earnings
contribution are as follows:
Co. A Co. B Total
Net asset contribution 300,000 400,000 700,000
Estimated annual earnings
Contribution 50,000 80,000 130,000

Stock holder of the two companies agreed that a single class of stock be issued that
their contribution be measured by net assets plus allowances for a goodwill, and that
10% considered as a normal rate of return ,Earnings in excess of the normal rate of
return sales be capitalized at 20% in the calculating goodwill , it was also agreed that
the authorized capital stock of the new corporation shall be 20,000 shares with a par
value of 100 a shares.

Determine: (1) The amount of goodwill credited to Co. A; and (2) the total
contribution of Co. B

(1) 100,000; (2) 400,000

1) 150,000; (2) 500,000

1)100,000; (2) 600,000

(1) 200,000; (2) 600,000


General Feedback
Note: this is an old structure
problem
Companies A and B A Co. B Co.
Net Asset Contribution 300,000.00 400,000.00
Multiply by: Normal Rate of Return 0.10 0.10
Normal Rate of Return 30,000.00 40,000.00
Estimated Annual Earnings 50,000.00 80,000.00
Estimated Excess Earnings 20,000.00 40,000.00
Divide by: Capitalization rate 0.20 0.20
Goodwill 100,000.00 200,000.00
Net Asset Contribution 300,000.00 400,000.00
Total Contribution 400,000.00 600,000.00
On August 31, Year 2, Wood Corp. issued 100,000 shares of its $20 par value
common stock for the net assets of Pine, Inc., in a business combination accounted for
by the acquisition method. The market value of Wood’s common stock on August 31
was $36 per share. Wood paid a fee of $160,000 to the consultant who arranged this
acquisition. Costs of registering and issuing the equity securities amounted to
$80,000. No goodwill was involved in the purchase. What amount should Wood
capitalize as the cost of acquiring Pine’s net assets?

$3,600,000

$3,680,000

$3,760,000

$3,840,000
General Feedback
In a business combination accounted for as an acquisition, the fair market value of the net
assets is used as the valuation basis for the combination. In this case, the net assets of the
subsidiary have an implied fair market value of $3,600,000 which is the value of the
common stock issued to Pine’s shareholders (100,000 shares × $36). The direct cost of
acquisition should not be included as part of the cost of a company acquired, and the cost
of registering equity securities should be a reduction of the issue price of the securities
(i.e., additional paid-in capital). Thus, the $160,000 paid for a consultant who arranged
the acquisition should be expensed, and the $80,000 cost for registering and issuing the
equity securities should be treated as a reduction of additional paid-in capital. Answer (a)
is correct because the total amount to be capitalized is $3,600,000.

On 1 January Year 1, Vinculum acquired 75% of Pathways’ equity shares by


means of an immediate share exchange of two shares in Vinculum for five shares in
Pathways. The fair value of Vinculum and Pathways’ shares on 1 January Year 1 were
4.00 and 3.00 respectively. In addition to the share exchange, Vinculum will make a
cash payment of 1.32 per acquired share, deferred until 1 January Year 2. Vinculum
has not recorded any of the consideration for Pathways in its financial statements.
Vinculum’s cost of capital is 10% per annum.
The summarized statements of financial position of the two companies as at 30 June
Year 1 are:
Vinculum Pathways
Assets
Non-current assets (note (ii))
Property, plant and equipment 55,000 28,600
Financial asset equity investments (note 11,500 6,000
(v))
66,500 34,600
Current assets
Inventory (note (iv)) 17,000 15,400
Trade receivables (note (iv)) 14,300 10,500
Bank 2,200 1,600
33,500 27,500
Total assets 100,000 62,100
Equity and liabilities
Equity
Equity shares of P1 each 20,000 20,000
Other component of equity 4,000 nil
Retained earnings – at 1 July 2014 26,200 14,000
Retained earnings – for year ended 30 24,000 10,000
June Year 1
74,200 44,000
Current liabilities (note (iv)) 25,800 18,100
Total equity and liabilities 100,000 62,100

The following information is relevant:


(i.) Pathways’ business is seasonal and 60% of its annual profit is made in the period 1
January to 30 June each year.
(ii.) At the date of acquisition, the fair value of Pathways’ net assets was equal to their
carrying amounts with the following exceptions:
a. An item of plant had a fair value of P2 million below its carrying value.
At the date of acquisition it had a remaining life of two years.
b. The fair value of Pathways’ investments was P7 million (see also note
(v)).
c. Pathways owned the rights to a popular mobile (cell) phone game. At
the date of acquisition, a specialist valuer estimated that the rights were worth
P12 million and had an estimated remaining life of five years.
(iii.) Following an impairment review, consolidated goodwill is to be written down by P3
million as at 30 June Year 1.
(iv.) (iv) Vinculum sells goods to Pathways at cost plus 30%. Pathways had P1·8 million
of goods in its inventory at
30 June Year 1 which had been supplied by Vinculum. In addition, on 28 June Year 1,
Vinculum processed the sale of P800,000 of goods to Pathways, which Pathways did not
account for until their receipt on 2 July Year 1. The in-transit reconciliation should be
achieved by assuming the transaction had been recorded in the books of Pathways before
the year end. At 30 June Year 1, Vinculum had a trade receivable balance of P2·4 million
due from Pathways which differed to the equivalent balance in Pathways’ books due to
the sale made on 28 June Year 1.
(v.) At 30 June Year 1, the fair values of the financial asset equity investments of
Vinculum and Pathways were P13·2 million and P7·9 million respectively.
(vi.) Vinculum’s policy is to value the non-controlling interest at fair value at the date of
acquisition. For this purpose, Pathways’ share price at that date is representative of the
fair value of the shares held by the non-controlling interest.
How much is the Non-Controlling Interest to be presented in the December 31, Year 1
consolidated statement of financial position?

17,500 16,550 15,800 15,000


General Feedback
Non-controlling interest
Fair value on acquisition (w (i)) 15,000
Post-acquisition profit (3,200 x 25% (w (iii)) 800
–––––––
15,800

Best Company has gained control over the operations of Cure Corporation by acquiring
85% of its outstanding capital stock for 2,580,000. This amount includes a control
premium of 30,000. Acquisition expenses, direct and indirect, amounted to 83,000 and
42,000 respectively.
Best Cure
Book Book Fair
Value Value Value
Cash 3,541,500 128,000
Accounts receivable 300,000 325,000
Inventories 550,000 360,000
Prepaid expenses 148,500 125,000
Land 2,350,000 879,000
Building 1,560,000 558,000
Equipment 300,000 185,000
Goodwill - 300,000
Total assets 8,750,000 2,860,000

Accounts payable 675,000 253,000


Notes payable 1,400,000 730,000
Capital stock, 50 par 3,400,000 800,000
Additional paid in 1,575,000 600,000
capital
Retained earnings 1,700,000 477,000
Total equities 8,750,000 2,860,000

The following was ascertained on the date of acquisition for Cure Corporation:
· The value of receivables and equipment has decreased by 25,000 and 14,000
respectively.
· The fair value of inventories is now P436,000 whereas the value of land and
building has increased by 471,000 and P107,000 respectively.
There was an unrecorded accounts payable amounting to 27,000 and the fair value of
notes is 738,000.
Compute for the following balances to be presented in the consolidated statement of
financial position at the date of business combination:
Total assets
9,875,000 10,093,000 10,112,000 9,215,000
General Feedback
10,093,000

Michangelo Co. paid $100,000 in fees to its accountants and lawyers in acquiring Florence
Company. Michangelo will treat the $100,000 as
an expense for the current year.

P Corporation used debentures with a par value of 610,000 to acquire 100 percent of the net
assets of S Company on January 1, Year 1 and S Company is dissolved. On that date, the fair
value of the bonds issued by P Corp. was 564,000, and the following balance sheet data were
reported by S Co.:
Balance Sheet Item Historical cost Fair value
Cash and Receivables 55,000 50,000
Inventory 105,000 200,000
Land 60,000 100,000
Plant and Equipment 400,000 300,000
Less: Accumulated Depreciation ( 150,000)
Goodwill 10,000
Total assets 480,000
Accounts Payable 50,000 50,000
Common Stock 100,000
Additional Paid-in Capital 60,000
Retained Earnings 270,000
Total Liabilities and Equities 480,000

P Corporation incurred an out of pocket expenses of 20,000. How much goodwill is to be


recognized on the books of P as a result of the business combination?

0
10,000
30,000

36,000
General Feedback
The result is a gain of P36,000, not goodwill
The answer is 0

On January 1, Year 1, Michi Corp. purchased 75% of the common stock of Maru Corp.
Separate balance sheet data for the companies at the combination date are given below:
Michi Maru
Cash 84,000 721,000
Trade Receivable 504,000 91,000
Merchandise Inventory 462,000 133,000
Land 273,000 112,000
Plant Assets 2,450,000 1,050,000
Accumulated Depreciation (840,000) (210,000)
Investment in Maru 1,372,000
Total Assets 4,305,000 1,897,000

Accounts Payable 721,000 497,000


Capital Stock 2,800,000 1,050,000
Retained Earnings 784,000 350,000
Total Equities 4,305,000 1,897,000

At the date of combination the book values of Maru net assets was equal to the fair value of
the net assets except for Maru’s inventory which has a fair value of 210,000.
On the date of acquisition in the consolidated balance sheet, How much is the total assets?

3,533,250 4,984,000 5,171,250 6,543,250


General Feedback
Fair value of net assets of Acquiree
Total Assets FV 1,974,000.00
Total Liabilities FV (497,000.00) 1,477,000.00

Consideration Transferred 1,372,000.00


Non-controlling Interest 369,250.00 1,741,250.00

Goodwill 264,250.00
Total Assets - Maru (Acquiree) - FV 1,974,000.00
Total Assets - Michi (Acquirrer) [excluding Inv. In Maru] 2,933,000.00
Consolidated Total Assets 5,171,250.00

On January 1, Year 1, Polk Corp. and Strass Corp. had condensed balance sheets as
follows:
Polk Strass
Current assets $ 70,000 $20,000
Noncurrent assets 90,000 40,000
Total assets $160,000 $60,000
Current liabilities 30,000 10,000
Long-term debt 50,000 --
Stockholders’ equity 80,000 50,000
Total liabilities and stockholders’ equity 160,000 60,000

On January 2, Year 1, Polk borrowed $60,000 and used the proceeds to purchase 90% of
the outstanding common shares of Strass. This debt is payable in ten equal annual
principal payments, plus interest, beginning December 30, Year 1. The excess cost of the
investment over Strass’ book value of acquired net assets should be allocated 60% to
inventory and 40% to goodwill. On January 1, Year 1, the fair
value of Polk shares held by noncontrolling parties was $10,000.

On Polk’s January 2, Year 1 consolidated balance sheet


Noncurrent liabilities should be

$115,000
 $109,000
$104,000 $ 55,000
General Feedback
In the consolidated balance sheet, the parent company’s “investment in subsidiary”
account should be eliminated and replaced by the assets and liabilities of the subsidiary.
Therefore, the consolidated balance sheet should include the noncurrent liabilities of both
companies, plus the noncurrent portion of the debt incurred on 1/2/11 ($60,000 – $6,000
= $54,000).

Noncurrent liabilities—Polk $ 50,000


Noncurrent liabilities—Strass 0
Noncurrent portion of new debt 54,000
Total $104,000

On January 1, Year 1, Polk Corp. and Strass Corp. had condensed balance sheets as follows:
Polk Strass
Current assets $ 70,000 $20,000
Noncurrent assets 90,000 40,000
Total assets $160,000 $60,000
Current liabilities 30,000 10,000
Long-term debt 50,000 --
Stockholders’ equity 80,000 50,000
Total liabilities and stockholders’ equity 160,000 60,000

On January 2, Year 1, Polk borrowed $60,000 and used the proceeds to purchase 90% of the
outstanding common shares of Strass. This debt is payable in ten equal annual
principal payments, plus interest, beginning December 30, Year 1. The excess cost of the
investment over Strass’ book value of acquired net assets should be allocated 60% to inventory
and 40% to goodwill. On January 1, Year 1, the fair
value of Polk shares held by noncontrolling parties was $10,000.

On Polk’s January 2, Year 1 consolidated balance sheet


Current assets should be
$ 90,000 $ 99,000 $100,000 $102,000
General Feedback
The cost of the investment is $60,000. The fair value of the non-controlling interest,
$10,000, is added to the cost of the investment of $60,000, to get the fair value of the net
assets of the business, $70,000. Next, the book value ($50,000) is subtracted from the fair
value to arrive at the differential of $20,000 ($70,000 – $50,000), which represents the
amount used to write up undervalued assets and recognize goodwill. Inventory is
increased by 60% of the $20,000, or $12,000. Therefore, current assets should be reported
at $102,000, as calculated below.
Current assets—Polk $70,000
Current assets—Strass 20,000
Excess allocated to inventory 12,000
Total current assets $102,000

A business combination is accounted for appropriately as an acquisition. Which of the


following should be deducted in determining the combined corporation’s net income
for the current period?
Direct costs General expenses
of acquisition related to acquisition

Yes No

Yes Yes

No Yes

No No
General Feedback
The direct costs of acquisition should be an expense of the period in a business combination
accounted for by the acquisition method. General expenses related to the acquisition are also
deducted as incurred in determining the combined corporation’s net income for the current
period. YES YES
When does the measurement period end for a business combination in which there was
incomplete accounting information on the date of acquisition? When the acquirer
receives the information or one year from the acquisition date, whichever occurs earlier.
On the final date when all contingencies are resolved. Thirty days from the date of
acquisition. At the end of the reporting period in the year of acquisition.
General Feedback
When the acquirer receives the information or one year from the acquisition date,
whichever occurs earlier.

On April 1, year 1, Parson Corp. purchased 80% of the outstanding stock of Sloan Corp.
for 700,000 cash. Parson determined that the fair value of the net identifiable assets was
800,000 on the date of acquisition. The fair value of Sloan’s stock at date of acquisition
was 18 per share. Sloan had a total of 50,000 shares of stock issued and outstanding prior
to the acquisition. What is the amount of goodwill that should be recorded by Parson at
date of acquisition?
0
60,000 80,000 120,000
General Feedback
80,000

On June 30, 2018, Wyler Corporation acquires Boston Corporation in a transaction


properly accounted for as a business acquisition. At the time of the acquisition, some of
the information for valuing assets was incomplete. How should Corporation Wyler,
account for the incomplete information in preparing its financial statements immediately
after the acquisition? Do not record the uncertain items until complete information is
available. Record a contra account to the investment account for the amounts
involved. Record the uncertain items at the book value of the acquiree. Record the
uncertain items at a provisional amount measured at the date of acquisition.
General Feedback
Record the uncertain items at a provisional amount measured at the date of acquisition.

On January 2, Year 2, PG Corporation sold equipment costing P100,000 with


accumulated depreciation of P25,000 to its wholly-owned subsidiary; SM Inc. the selling
price was P90,000. PG was depreciating the equipment on the straight-line method
inventory over twenty years with salvage value. SM continued this depreciation. What are
the cost and accumulated depreciation, respectively, of this equipment in the December
31, Year 2 consolidated statement of financial position? P75,000 and P3,750
P90,000 and P4,500 P90,000 and P29,500 P100,000 and P30,000
General Feedback
P100,000 and P30,000

Penny Company owns an 80% controlling interest in the Sandy’s Company. Sandy
regularly sells merchandise to Penny, which then sold to outside parties. The gross profit
on all such sales is 40%. On January 1, Year 2, Penny sold land and a building to Sandy.
The value of the parcel is 20% to land and 80% to structures. Pertinent data for the
companies is summarized in the next page.

Penny Sandy
Internally generated net income, Year 2 520,000 250,000
Internally generated net income, Year 3 340,000 235,000
Intercompany merchandise sales, Year 2 100,000
Intercompany merchandise sales, Year 3 120,000
Intercompany inventory, December 31,
Year 2 15,000
Intercompany inventory, December 31,
Year 3 20,000
Cost of real estate sold on January 1, Year
2 600,000
Sales price of real estate on January 1, Year
2 800,000
Depreciable life of building 20 yrs
For Year 2, what is the consolidated comprehensive income attributable to controlling
interest?
523,200 525,000 625,000 532,500
General Feedback
523,200

On January 1, Year 1 SST Company purchased a computer with an expected life of 5


years. On January 1, Year 3 SST Company sold the computer to PMN corporation and
recorded the following entry:
Cash P39, 000
Accumulated 16, 000
Depreciation
Computer Equipment 40, 000
Gain on sale of
15, 000
equipment

PMN Corporation holds 60% of the voting shares of SST Company. SST Company and
PMN Corporation reported income from its own operations of P45, 000 and P85, 000 for
Year 3 respectively. There is no change in the estimated life of the equipment as a result
of intercompany sale.

How much is the income attributable to the Non-Controlling Interest for Year 3?

12,000 14,000 18,000 21,000


General Feedback
PMN Corporation and SST Company
EHP NCI Consolidated
Profit - Parent (own operation) 85,000 85,000
Profit - Subsidiary 27,000 18,000 45,000
Intercompany sale of PPE
Eliminate Gain (9,000) (6,000) (15,000)
Amortization of
gain 3,000 2,000 5,000
Total 106,000 14,000 120,000
#12 #13
14,000

Port, Inc. owns 100% of Salem, Inc. On January 1, 2018, Port sold Salem delivery
equipment at a gain. Port had owned the equipment for two years and used a five-year
straight-line depreciation rate with no residual value. Salem is using a three-year straight-
line depreciation rate with no residual value for the equipment. In the consolidated
income statement, Salem’s recorded depreciation expense on the equipment for 2018 will
be decreased by

20% of the gain on sale. 33 1/3% of the gain on sale. 50% of the gain on sale.
100% of the gain on sale
General Feedback
33 1/3% of the gain on sale.
On January 1, Year 1 SST Company purchased a computer with an expected life of 5
years. On January 1, Year 3 SST Company sold the computer to PMN corporation and
recorded the following entry:
Cash P39, 000
Accumulated
16, 000
Depreciation
Computer Equipment 40, 000
Gain on sale of
15, 000
equipment

PMN Corporation holds 60% of the voting shares of SST Company. SST Company and
PMN Corporation reported income from its own operations of P45, 000 and P85, 000 for
Year 3 respectively. There is no change in the estimated life of the equipment as a result
of intercompany sale.
What is the consolidated total comprehensive income attributable to parent for Year 3?

P103, 000 P106, 000 - correct


P112, 000 P130, 000
General Feedback
PMN Corporation and SST Company
EHP NCI Consolidated
Profit - Parent (own operation) 85,000 85,000
Profit - Subsidiary 27,000 18,000 45,000
Intercompany sale of PPE
Eliminate Gain (9,000) (6,000) (15,000)
Amortization of
gain 3,000 2,000 5,000
Total 106,000 14,000 120,000
#13
#12
BigBang Company owns an 80% controlling interest in Sheldon Company. Sheldon
regularly sells merchandise to BigBang, which then sells to outside parties. The gross
profit on all such sales is 40%. On January 1, Year 1, BigBang sold land and a
building to Sheldon. The value of the parcel is 20% to land and 80% to structures.
Pertinent data for the companies is summarized below.
BigBang Sheldon
Internally generated net income, Year 2 340, 000 235, 000
Internally generated net income, Year 1 P520, 000 P250, 000
Intercompany merchandise sales, Year
120, 000
2
Intercompany merchandise sales, Year
100, 000
1
Intercompany inventory, December 31,
20, 000
Year 2
Intercompany inventory, December 31,
15, 000
Year 1
Cost of real estate sold on January 1,
600, 000
Year 1
Sales price of real estate on January 1,
800, 000
Year 1
Depreciable life of building 20 years.

For Year 2, what is the consolidated comprehensive income attributable to controlling


interest?

453, 400 534, 400 543, 000 543, 400


General Feedback
BigBang Company
EHP
Profit - Parent (own operation) 340,000
Profit - Subsidiary 188,000 (235,000*80%)
Intercompany sale - Inventory (upstream)
Unrealized gross profit (6,400) (20,000*40%*80%)
Realized gross profit 4,800 (15,000*40%*80%)
Intercompany sale - PPE (downstream)
PPE - amortization of [((800,000-
gain 8,000 600,000)*80%)/20]
Profit - EHP 534,400

AtC Industries manufactures heavy equipment used in construction and excavation.


On January 3, Year 1 AtC sold a piece of equipment from its inventory that cost
P180,00 to its 60% owned subsidiary, JlB Corporation, at AtC’s standard price is
twice its cost. Jlb is depreciating the equipment over six years using straight-line
depreciation and no salvage value.

How much depreciation must be eliminated in preparing the consolidated financial


statement?
60,000 30,000 36,000 0
General Feedback
Amortization of the Markup on sale
(180,000 / 6) 30,000

*note: Eliminate the Sale of inventory; eliminate the corresponding


Markup ("gain")
The true depreciation must be based on the CV of the selling
affiliate
On January 1, 2018, Poe Corp. sold a machine for 900,000 to Saxe Corp., its wholly
owned subsidiary. Poe paid 1,100,000 for this machine, which had accumulated
depreciation of 250,000. Poe estimated a 100,000 salvage value and depreciated the
machine on the straight-line method over twenty years, a policy which Saxe
continued. In Poe’s December 31, 2018 consolidated balance sheet, this machine
should be included in cost and accumulated depreciation as
1) Cost
2) Accumulated depreciation

1) 1,100,000 2) 300,000

1) 1,100,000 2) 290,000

1) 900,000 2) 40,000

1) 850,000 2) 42,500
General Feedback
1) 1,100,000 2) 300,000

Penny Company owns an 80% controlling interest in the Sandy’s Company. Sandy
regularly sells merchandise to Penny, which then sold to outside parties. The gross profit
on all such sales is 40%. On January 1, Year 2, Penny sold land and a building to Sandy.
The value of the parcel is 20% to land and 80% to structures. Pertinent data for the
companies is summarized in the next page.

Penny Sandy
Internally generated net income, Year 2 520,000 250,000
Internally generated net income, Year 3 340,000 235,000
Intercompany merchandise sales, Year 2 100,000
Intercompany merchandise sales, Year 3 120,000
Intercompany inventory, December 31,
Year 2 15,000
Intercompany inventory, December 31,
Year 3 20,000
Cost of real estate sold on January 1, Year
2 600,000
Sales price of real estate on January 1, Year
2 800,000
Depreciable life of building 20 yrs
For Year 3, what is the consolidated comprehensive income attributable to controlling
interest?
534,400 543,000 453,400 543,400
General Feedback
534,400

On January 1, Year 1, Harny Company purchased an Equipment with an expected


useful life of 5 years and scrap value of P8,000. On January 1, Year 3, Harny
Company sold the truck to Jazel Company and recorded the following entry:
Debit Credit
Cash 50,000
Accumulated 18,000
depreciation
Truck 53,000
Gain on sale of 15,000
truck

Jazel holds 60% of Harny's voting shares and considered the equipment to have a
remaining useful life of 4 years. Harny reported net income of P55,000, and Jazel
reported separate net income of P98,000 for Year 3.
In preparing the consolidated financial statements for Year 3, depreciation expense
recorded by the Jazel will be:
Increased by 9,000 Reduced by 6,667 Reduced by 1,500 Increased by 5,000
General Feedback
Harny Company
Recorded depreciation 10,500 ((50,000-8,000)/4)
Should be depreciation 9,000 ((53,000-8,000)/5)
Adjustment (reduction in
depreciation) 1,500

Pact acquired 80% of the equity shares of Sact on 1 July Year 1, paying P3.00
for each share acquired. This represented a premium of 20% over the market price of
Sact’s shares at that date. Sact’s shareholders’ funds (equity) as at 31 March Year 2 were:

Equity shares of P1 each 100,000


Retained earnings at 1 April Year 1 80,000
Profit for the year ended 31 March 40,000 120,000
Year 2
220,000

The only fair value adjustment required to Sact’s net assets on consolidation was a
P20,000 increase in the value of its land. Pact’s policy is to value non-controlling
interests at fair value at the date of acquisition. For this purpose the market price of
Sact’s shares at that date can be deemed to be representative of the fair value of the
shares held by the non-controlling interest
How much is the profit attributable to the Controlling Interest?

40,000 32,000 24,000 8,000


General Feedback
Pact and Sact
Profit reported by subsidiary 40,000.00
Multiply by number of months with control 9/12
Multiply by percentage of
ownership 80%
Profit attributable to Controlling
Interest 24,000.00
*It was assumed that no further equity transactions transpired from the acquisition date
since no data is available.

Zuma Corporation and its subsidiary reported consolidated net income of P320,000 for
the year ended December 31, Year 1. Zuma owns 80 percent of the common shares of its
subsidiary, acquired at book value. Noncontrolling interest was assigned income of
P30,000 in the consolidated income statement for Year 1. What is the amount of separate
operating income reported by Zuma for the year? P170,000 P150,000 P120,000
P200,000
General Feedback
Zuma Corporation
Consolidated Income 320,000.00
Less: Adjusted Subsidiary Income
(30,000/20%) (150,000.00)
Parent company's adjusted separate income 170,000.00
Pact acquired 80% of the equity shares of Sact on 1 July Year 1, paying P3.00
for each share acquired. This represented a premium of 20% over the market price of
Sact’s shares at that date. Sact’s shareholders’ funds (equity) as at 31 March Year 2 were:

Equity shares of P1 each 100,000


Retained earnings at 1 April Year 1 80,000
Profit for the year ended 31 March Year 2 40,000 120,000
220,000
The only fair value adjustment required to Sact’s net assets on consolidation was a
P20,000 increase in the value of its land. Pact’s policy is to value non-controlling
interests at fair value at the date of acquisition. For this purpose the market price of
Sact’s shares at that date can be deemed to be representative of the fair value of the
shares held by the non-controlling interest.

Sub Company sells all its output at 20 percent above cost to Par Corporation. Par
purchases all its inventory from Sub. The incomes reported by the companies over the
past three years are as follows:

Year Sub Company’s Net Income Par Corporation’s Operating Income


Year 1 150,000 225,000
Year 2 135,000 360,000
Year 3 240,000 450,000

Sub Company sold inventory for P300,000, P262,500 and P337,500 in the years Year 1,
Year 2, and Year 3 respectively. Par Company reported ending inventory of P105,000,
P157,500 and P180,000 for Year 1, Year 2, and Year 3 respectively. Par acquired 70
percent of the ownership of Sub on January 1, Year 1, at underlying book value. The fair
value of the noncontrolling interest at the date of acquisition was equal to 30 percent of
the book value of Sub Company.

What will be the income assigned to controlling interest for Year 2?

P448,375 P495,000 P486,250 P615,37


General Feedback
Profit - Par, Year 2 360,000.00
Share in Profit - Sub, Year 2 94,500.00
Share in RGP in beginning Inventory
(105,000/120%*20%*70%) 12,250.00
Share in DGP in Ending Inventory
(157,500/120%*20%*70%) (18,375.00)
Profit - Equity Holders of Parent 448,375.00

Roland Company acquired 100 percent of Garros Company's voting shares in Year 1.
During Year 2, Garros purchased tennis equipment for P30,000 and sold them to Roland
for P55,000. Roland continues to hold the items in inventory on December 31, Year 2.
Sales for the two companies during Year 2 totaled P655,000, and total cost of goods sold
was P420,000. Which of the following observations will be true if no adjustment is made
to eliminate the intercorporate sale when a consolidated income statement is prepared for
Year 2? Sales would be overstated by P30,000 Cost of goods sold will be
understated by P25,000 Net income will be overstated by P25,000 Consolidated net
income will be unaffected
General Feedback
Roland Company
If NO adjustment is made for the inter-company
sales:
Statement A is
Overstatement in Sales 55,000.00 FALSE
Overstatement in Cost of Goods Statement B is
Sold 55,000.00 FALSE
Overstatement in Ending Inventory 25,000.00
Statement C is
Overstatement in Profit 25,000.00 TRUE

Pact acquired 80% of the equity shares of Sact on 1 July Year 1, paying P3.00
for each share acquired. This represented a premium of 20% over the market price of
Sact’s shares at that date. Sact’s shareholders’ funds (equity) as at 31 March Year 2 were:

Equity shares of P1 each 100,000


Retained earnings at 1 April Year 1 80,000
Profit for the year ended 31 March 40,000 120,000
Year 2
220,000

The only fair value adjustment required to Sact’s net assets on consolidation was a
P20,000 increase in the value of its land. Pact’s policy is to value non-controlling
interests at fair value at the date of acquisition. For this purpose the market price of
Sact’s shares at that date can be deemed to be representative of the fair value of the
shares held by the non-controlling interest.

What would be the carrying amount of the non-controlling interest of Sact in


the consolidated statement of financial position of Pact as at 31 March Year 2?

P58,00 P56,000 P54,000 P50,000


General Feedback
Pact and Sact
Equity shares, March 1, Year 2 (# of
shares) 100,000.00
Multiply by: NCI percentage 20%
NCI held shares 20,000.00
Multiply by: FV/Share (P3 / 120%) 2.50
NCI - Net asset, at acquisition* 50,000.00
Share in profit (40,000 * 9/12 * 20%) 6,000.00
Share in dividends -
NCI - Net asset, March 31, Year 2 56,000.00

On January 1, Year 1, Wilhelm Corporation acquired 90 percent of Kaiser Company's


voting stock, at underlying book value. The fair value of the noncontrolling interest was
equal to 10 percent of the book value of Kaiser at that date. Wilhelm uses the equity
method in accounting for its ownership of Kaiser. On December 31, Year 2, the trial
balances of the two companies are as follows:
Wilhelm Corporation Kaiser Company
Debit Credit Debit Credit
Current Assets 200,000 140,000
Depreciable Assets 350,000 250,000
Investment in Kaiser 162,000
Company Stock
Depreciation Expense 27,000 10,000
Other Expenses 95,000 60,000
Dividends Declared 20,000 10,000
Accumulated Depreciation 118,000 80,000
Current Liabilities 100,000 80,000
Long-Term Debt 100,000 50,000
Common Stock 100,000 50,000
Retained Earnings 150,000 100,000
Sales 250,000 110,000
Income from Subsidiary 36,000
Based on the preceding information, what amount would be reported as retained earnings
in the consolidated balance sheet prepared at December 31, Year 2?
314,000 294,000 150,000 424,000
General Feedback
Wilhelm Corporation
Retained Earnings - Parent 150,000.00
Profit - Parent
Sales 250,000.00
Depreciation Expense (27,000.00)
Other expenses (95,000.00) 128,000.00
Dividends Declared - Parent (20,000.00)
Retained Earnings - Parent, Ending 258,000.00
Parent's share in Subsidiary Profit -
2009 36,000.00
Retained Earnings - Equity Holders
of Parent 294,000.00
Parker Corp. owns 80% of Smith Inc.’s common stock. During 2017, Parker sold
Smith 250,000 of inventory on the same terms as sales made to third parties. Smith
sold all of the inventory purchased from Parker in 2017. The following information
pertains to Smith and Parker’s sales for 2017:
Parker Smith
Sales 1,000,000 700,000
Cost of sales 400,000 350,000
600,000 350,000

What amount should Parker report as cost of sales in it s 2017 consolidated income
statement?
750,000 680,000 500,000 430,000
General Feedback
500,000

Wilmslow acquired 80% of the equity shares of Zeta on 1 April Year 1 when Zeta’s
retained earnings were P200,000. During the year ended 31 March Year 2, Zeta
purchased goods from Wilmslow totalling P320,000. At 31 March Year 2, one quarter
of these goods were still in the inventory of Zeta. Wilmslow applies a mark-up on cost
of 25% to all of its sales. At 31 March Year 2, the retained earnings of Wilmslow and
Zeta were P450,000 and P340,000 respectively.

What would be the amount of retained earnings in Wilmslow’s consolidated statement


of financial position as at31 March Year 2?
P706,000 P546,000 P542,000 P498,0
General Feedback
Wilmslow
Retained Earnings, end - Parent - separate FS 450,000.00
Unrealized gross profit on Downstream sales
[(320,000/4)/125%)*25%] (16,000.00)
Share in adjusted profit of
Subsidiary
Subsidiary reported profit [(340,000-
200,000)*80%] 112,000.00
Consolidated Retained Earnings 546,000.00

On January 1, Year 1, Wilhelm Corporation acquired 90 percent of Kaiser Company's


voting stock, at underlying book value. The fair value of the noncontrolling interest was
equal to 10 percent of the book value of Kaiser at that date. Wilhelm uses the equity
method in accounting for its ownership of Kaiser. On December 31, Year 2, the trial
balances of the two companies are as follows:
Wilhelm Corporation Kaiser Company
Debit Credit Debit Credit
Current Assets 200,000 140,000
Depreciable Assets 350,000 250,000
Investment in Kaiser 162,000
Company Stock
Depreciation Expense 27,000 10,000
Other Expenses 95,000 60,000
Dividends Declared 20,000 10,000
Accumulated Depreciation 118,000 80,000
Current Liabilities 100,000 80,000
Long-Term Debt 100,000 50,000
Common Stock 100,000 50,000
Retained Earnings 150,000 100,000
Sales 250,000 110,000
Income from Subsidiary 36,000
Based on the preceding information, what amount would be reported as total
stockholder's equity in the consolidated balance sheet at December 31, Year 2?
P412,000 P394,000 P542,000 P348,000
General Feedback
Common Stock 100,000.00
Retained Earnings - Equity Holders
of Parent 294,000.00
Non-Controlling Interest
Common Stock - Subsidiary 50,000.00 *Note: Since no
Retained Earnings, Beg - additional information is
Subsidiary 100,000.00 provided as to equity
Subsidiary Profit - Current Year 40,000.00 transaction after
Subsidiary Dividend Declared (10,000.00) acquisition, it is assumed
Subsidiary Net Assets, ending 180,000.00 that no subsequent
issuance or acquisition of
shares transpired
Mutiply by: NCI percentage 0.10 18,000.00 subsequent to acquisition
Total Stockholders' Equity 412,000.00

During 2017, Pard Corp. sold goods to its 80%-owned subsidiary, Seed Corp. At December 31,
2017, one-half of these goods were included in Seed’s ending inventory. Reported 2017 selling
expenses were 1,100,000 and 400,000 for Pard and Seed, respectively. Pard’s selling expenses
included 50,000 in freight-out costs for goods sold to Seed.
What amount of selling expenses should be reported in Pard’s 2017 consolidated income
statement?

1,500,000 1,480,000 1,475,000 1,450,000


General Feedback
1,450,000

On January 1, Year 2, Carlito Company acquired 80% interests in Harries Company for
P2,000,000 cash. The stockholder’s equity of Harries at the time of acquisition is
P1,875,000. On January 1, Year 2, NCI is measured at its implied fair value. The excess
of cost over books value of interest acquired is allocated to the following assets:

Inventories P100, 000 (sold in Year 2)


Building P200, 000 (5- year remaining
life)

During Year 2, Harries Company reported total comprehensive income of P500,000 and
paid dividend for P100,000.
What is the NCI in net assets of subsidiary on December 31, Year 2?
General Feedback
P552,000

On January 1, Year 2, Carlito Company acquired 80% interests in Harries Company for
P2,000,000 cash. The stockholder’s equity of Harries at the time of acquisition is
P1,875,000. On January 1, Year 2, NCI is measured at its implied fair value. The excess
of cost over books value of interest acquired is allocated to the following assets:

Inventories P100, 000 (sold in Year 2)


Building P200, 000 (5- year remaining
life)

During Year 2, Harries Company reported total comprehensive income of P500,000 and
paid dividend for P100,000.
What was the fair value of NCI on January 1, Year 2?

P500,000 P375,000 P525,000 P400,000


General Feedback
P500,000
A subsidiary was acquired for cash in a business combination on January 1, 2017. The
consideration given exceeded the fair value of identifiable net assets. The acquired
company owned equipment with a market value in excess of the carrying amount as of the
date of combination. A consolidated balance sheet prepared on December 31, 2017,
would Report the unamortized portion of the excess of the market value over the
carrying amount of the equipment as part of goodwill. Report the unamortized portion
of the excess of the market value over the carrying amount of the equipment as part of
plant and equipment. Report the excess of the market value over the carrying amount
of the equipment as part of plant and equipment. Not report the excess of the market
value over the carrying amount of the equipment because it would be expensed in the
year of the acquisition
General Feedback
Report the unamortized portion of the excess of the market value over the carrying
amount of the equipment as part of plant and equipment.

On January 1, Year 2, Carlito Company acquired 80% interests in Harries Company for
P2,000,000 cash. The stockholder’s equity of Harries at the time of acquisition is
P1,875,000. On January 1, Year 2, NCI is measured at its implied fair value. The excess
of cost over books value of interest acquired is allocated to the following assets:

Inventories P100, 000 (sold in Year 2)


Building P200, 000 (5- year remaining
life)

During Year 2, Harries Company reported total comprehensive income of P500,000 and
paid dividend for P100,000.
What is the consolidated total comprehensive income attributable to parent on December
31, Year 2, if Carlito’s net income for Year 2 is P600,000?

876,000 888,000 808,000 948,000


General Feedback
808,000

On January 1, Year 2, Owen Corp. purchased all of Sharp Corp.’s common stock for
P1,200,000. On that date, the fair values of Sharp’s assets and liabilities equaled their
carrying amounts of P1,320,000 and P320,000, respectively. During Year 2, Sharp
paid cash dividends of P20,000. Selected information from the separate balance sheets
and income statements of Owen and Sharp as of December 31, Year 2, and for the
year then ended follows:
Owen Sharp
Balance sheet accounts
Investment in subsidiary 1,320,000 -
Retained earnings 1,240,000 560,000
Total stockholders’ equity 2,620,000 1,120,000
Income statement accounts
Operating income 420,000 200,000
Equity in earnings of Sharp 140,000 -
Net income 400,000 140,000
In Owen’s December 31, Year 2 consolidated balance sheet, what amount should be
reported as total retained earnings? P1,240,000 P1,360,000 P1,380,000
P1,800,000
General Feedback
P1,240,000

On January 1, 2017, Palm, Inc. purchased 80% of the stock of Stone Corp. for 4,000,000
cash. Prior to the acquisition, Stone had 100,000 shares of stock outstanding. On the date
of acquisition, Stone’s stock had a fair value of 52 per share. During the year Stone
reported 280,000 in net income and paid dividends of 50,000. What is the balance in the
noncontrolling interest account on Palm’s balance sheet on December 31, 2017?
1,000,000 1,040,000 1,086,000 1,096,000
General Feedback
1,086,000

Andrew owns 100% of the share capital of the following companies. The directors are
unsure of whether the investments should be consolidated.
In which of the following circumstances would the investment NOT be consolidated?
Andrew has decided to sell its investment in Alpha as it is loss-making; the directors
believe its exclusion from consolidation would assist users in predicting the group’s
future profits Beta is a bank and its activity is so different from the engineering
activities of the rest of the group that it would be meaningless to consolidate it
Gamma is located in a country where a military coup has taken place and seized full
ownership of all private entities Delta is located in a country where local accounting
standards are compulsory and these are not compatible with IFRS used by the rest of the
group
General Feedback
Gamma is located in a country where a military coup has taken place and seized full
ownership of all private entities
On January 1, Year 2, Carlito Company acquired 80% interests in Harries Company for
P2,000,000 cash. The stockholder’s equity of Harries at the time of acquisition is
P1,875,000. On January 1, Year 2, NCI is measured at its implied fair value. The excess
of cost over books value of interest acquired is allocated to the following assets:

Inventories P100, 000 (sold in Year 2)


Building P200, 000 (5- year remaining
life)

During Year 2, Harries Company reported total comprehensive income of P500,000 and
paid dividend for P100,000
How much goodwill (gain on acquisition) is reported in the consolidated statement of
financial position on 1/1/Year 2?

P325,000 P200,000 P(325,000) P(375,000)


General Feedback
P325,000

A subsidiary was acquired for cash in a business combination on January 1, Year 1. The
consideration given exceeded the fair value of identifiable net assets. The acquired
company owned equipment with a market value in excess of the carrying amount as of the
date of combination. A consolidated balance sheet prepared on December 31, Year 1,
would Report the unamortized portion of the excess of the market value over the
carrying amount of the equipment as part of plant and equipment Report the
unamortized portion of the excess of the market value over the carrying amount of the
equipment as part of goodwill Report the excess of the market value over the carrying
amount of the equipment as part of plant and equipment Not report the excess of the
market value over the carrying amount of the equipment because it would be expensed in
the year of the acquisition
General Feedback
Report the unamortized portion of the excess of the market value over the carrying
amount of the equipment as part of plant and equipment

Baduy Corp. owns 80 percent of the stock of Hiphop Company. At the end of Year 2,
Baduy Corp. and Hiphop Company reported the following partial operating results
and inventory balances:
Baduy Hiphop
Corp. Co.
Total sales 658,000 510,000
Sales to Hiphop Co. 140,000
Sales to Baduy Corp. 240,000
Profit 20,000
Operating Profit (excluding income from 70,000
Hiphop Co.)
Inventory, December 31, Year 2:
Purchases from Hiphop Co. 48,000
Purchases from Baduy Corp. 42,000

Baduy Corporation regularly prices its products at cost plus a 40 percent mark-up for
profit. Hiphop Company prices its sales at cost plus a 20 percent mark-up. The total
sales reported by Baduy and Hiphop include both intercompany sales and sales to
nonaffiliates.
The consolidated cost of sales for Year 2 must be:
790,000 770,000 535,000 496,333
General Feedback
baduy Corp
Cost of goods sold - Parent (658,000/140%) 470,000
Cost of goods sold - Subsidiary (510,000/120%) 425,000
(380,000
Intercompany Sale (140,000+240,000) )
Unrealized gross profit on ending inventory
[(48,000/120%)*20%
] (6,667)
[(42,000/140%)*40%
] (12,000)
Consolidated Cost of Goods Sold 496,333

Pat Corp. owns 80% for Sir. Inc. common stock. During Year 2, Pat sold Sir P250,000
of inventory on the same terms as sale made to third parties. Sir sold all of the
inventory purchased from Pat in Year 2. The following information pertains for Sir
and Pat’s sales for Year 2.

Pat Sir
P1,000,00
Sales P700,000
0
Cost of
400,000 350,000
sales
Gross profit P600,000 P350,000
What amount should Pat report as cost of sales in its Year 2 consolidated statement of
comprehensive income? P750,000 P680,000 P500,000 P430,000
General Feedback
P500,000

A subsidiary made sales of inventory to its parent at a profit this year. The parent, in turn,
sold all but 20 percent of the inventory to unaffiliated companies, recognizing a profit.
The amount that should be reported as cost of goods sold in the consolidated income
statement prepared for the year should be: the amount reported as intercompany sales
by the subsidiary the amount reported as intercompany sales by the subsidiary minus
unrealized profit in the ending inventory of the parent the amount reported as cost of
goods sold by the parent minus unrealized profit in the ending inventory of the parent
the amount reported as cost of goods sold by the parent
General Feedback
the amount reported as intercompany sales by the subsidiary minus unrealized profit in
the ending inventory of the parent

Planet Company acquired a 70% interest in the Star Company in 2016. For the year ended
December 31, 2017, Star reported net income of 80,000. During 2017, Planet sold
merchandise to Star for 10,000 at a profit of 2,000. The merchandise remained in Star’s
inventory at the end of 2017. For consolidation purposes what is the noncontrolling
interest’s share of Star’s net income for 2017? 23,400 24,000 24,600 26,000
General Feedback
24,000

Novy Corporation purchased at book value 70 percent of the ownership of Meiji


Corporation and 90 percent of the ownership of Cecille Corporation in Year 1. There
are frequent intercompany transfers among the companies. Activity relevant to Year 4
is presented below.
Transf Unsold
Producti er at End Year
Year Producer on Cost Buyer Price of Year Sold
Year Meiji Novy Year
3 Corp. 24,000 Corp. 30,000 10,000 4
Year Cecille Meiji Year
3 Corp. 60,000 Corp. 72,000 18,000 4
Year Novy Meiji Year
4 Corp. 15,000 Corp. 35,000 7,000 5
Year Meiji Cecille Year
4 Corp. 63,000 Corp. 72,000 12,000 5
Year Cecille 27,000 Novy 45,000 15,000 Year
4 Corp. Corp. 5

For the year ended December 31, Year 4, Novy Corporation reported P80,000 of
income from its separate operations (excluding income from intercorporate
investments). Meiji Corp. reported net income of P37,500, and Cecille Corporation
reported net income of P20,000.

Compute the amount reported as consolidated net income for Year 4.


P 117,900 P 116,750 P 142,50 P 96,750
General Feedback
Consolidate
Novy Corporation EHP d
Profit - Parent (Novy) - separate
operations 80,000.00 80,000.00
Profit - Subsidiary (Meiji) 26,250.00 37,500.00
Profit - Subsidiary (Cecille) 18,000.00 20,000.00
Intercompany Sales
RGP: Meiji - Novy
[10,000*20%] 1,400.00 2,000.00
RGP: Cecille - Meiji
[18,000/120%*20%] 2,700.00 3,000.00
UGP: Novy - Meiji
[7,000*57.14%] (3,999.80) (3,999.80)
UGP: Meiji - Cecille
[12,000*12.5%] (1,050.00) (1,500.00)
UGP: Cecille - Novy
[15,000*40%] (5,400.00) (6,000.00)
Total Profit 117,900.20 131,000.20
*the amount requested is "consolidated Profit", but amount included in
choices is "consolidated profit attributable to parent"
**kindly observe the computation for inter-
subsidiary sale

Parker Corp. owns 80% of Smith Inc.’s common stock. During 2017, Parker sold
Smith 250,000 of inventory on the same terms as sales made to third parties. Smith
sold all of the inventory purchased from Parker in 2017. The following information
pertains to Smith and Parker’s sales for 2017:
Parker Smith
Sales 1,000,000 700,000
Cost of sales 400,000 350,000
600,000 350,000
What amount should Parker report as cost of sales in it s 2017 consolidated income
statement?
750,000 680,000 500,000 430,000
General Feedback
500,000

OnSeptember 22, Year 2, Y Corporation purchased merchandise from an unaffiliated


foreign company for 10,000 units of the foreign company’s local currency. On that
date, the spot rate was P55. Y paid the bill in full on March 20, Year 3, when the spot
rate was P55.50. The spot rate was P56 on December 31, Year 2. What amount should
Yumi report as a foreign currency transaction loss in its income statement for the year
ended December 31, Year 2?
P0 P5,000 P10,000 P15,000
General Feedback
P10,000

CERTS Company, a Philippine Corporation, bought inventory from a supplier in


Japan on November 2, Year 1 for 50,000 yen, when the spot rate was P 0.4245. On
December 31, Year 2, the spot rate was P 0.4295. On January 15, 2012, CERTS
bought 50,000 yen at a spot rate of P 0.4250 and paid the invoice. How much should
CERTS report in its income statements for (1) Year 2 and (2) 2012 as foreign
exchange gain or (loss)

P250; (P225)

(P250); P225

P0; (P225)

P0; P220
General Feedback
(P250); P225

On July 1, Year 2, ACCENTURE Company lent P308,000 to a US supplier, evidenced


by an interest-bearing-note due on July 1, Year 3. The note is equivalent to $8,000 on
the loan date. The note principal was appropriately included at P328,000 in
ACCENTURE’s December 31, Year 2 balance sheet. The note was repaid to
ACCENTURE on July 1, Year 3. Due date when the exchange rate was P39 to $1. In
its income statement for the year Ended December 31, Year 3 what amount should
ACCENTURE company include as a foreign currency transaction gain or loss?

P0

P26,000 gain

P16,000 gain

P16,000 loss
General Feedback
P16,000 loss

Matthew, a money changer speculate in foreign currency as his business. On October 1,


Year 2, Matthew bought a 180-day forward contract to purchase 5, 000 FC at a forward
rate of FC1= P56.50 when the spot rate was P56.00. Other exchange rates were as
follows:

Forward Rate for March 31, Year 3


Spot Rate
Dec. 31, Year 2 P56.30 P56.60
Mar. 31, Year 3 56.32

The forex gain (loss) recognized by Matthew from this forward contract is:

P1,500 P(900) P500 P(10,000)


General Feedback
P(900)

A Pampanga food processor forecasts purchasing 300,000 pounds of soybean oil in May.
On February 20, the company acquires an option to buy 300,000 pounds of soybean oil in
May at a strike price of P1.60 per pound. Information regarding spot prices and option
values at selected dates is as follows:

February February March April20


20 28 31
Spot rate (market price) P1.61 P1.59 P1.62 P1.64
Per pound
Strike price (exercise 1.60 1.60 1.60 1.60
price)
Fair value of call option P3,800 P1,200 P6,800 P12,500

The company settled the option on April 20 and purchased 300,000 pounds of soybean oil
on May 3 at a spot price of P1.63 per pound. During May, the soybean oil was used to
produce food. One-half of the resulting food was sold in June. The change in the option’s
time value is excluded from the assessment of hedge effectiveness.

The cost of inventory that acquired on May 3:

P480,000 P489,000 P483,000 P492,000


General Feedback
P489,000

A Pampanga food processor forecasts purchasing 300,000 pounds of soybean oil in May.
On February 20, the company acquires an option to buy 300,000 pounds of soybean oil in
May at a strike price of P1.60 per pound. Information regarding spot prices and option
values at selected dates is as follows:

February February March April20


20 28 31
Spot rate (market price) P1.61 P1.59 P1.62 P1.64
Per pound
Strike price (exercise 1.60 1.60 1.60 1.60
price)
Fair value of call option P3,800 P1,200 P6,800 P12,500

The company settled the option on April 20 and purchased 300,000 pounds of soybean oil
on May 3 at a spot price of P1.63 per pound. During May, the soybean oil was used to
produce food. One-half of the resulting food was sold in June. The change in the option’s
time value is excluded from the assessment of hedge effectiveness.

The foreign exchange gain (or loss) on option contract (hedging instrument) on February
28:

OCI Earnings

(1,600) P400 (3,000) P400 3,000 (400) 0 0


General Feedback
(3,000) P400

Certain balance sheet accounts of a foreign subsidiary in Filam, Inc at December 31,
Year 2 have been translated into Philippine pesos as follows:

Current rate Historical Rate


Accounts receivable P120,000 P100,000
Prepaid insurance 55,000 50,000
Copyright 75,000 85,000

What was the total amount included in Filam’s December 31, Year 2 consolidated
balance sheet for the above accounts?
P255,000 P235,000 P240,000 P250,000
General Feedback
P250,000

A subsidiary of Benilan[1], Inc. located in a foreign country whose functional


currency is the foreign currency (which is not currency of a hyperinflationary
economy). The subsidiary acquires inventory on credit on November 1,Year 1, for
100,000 foreign currencies (FC) that is sold on January 17,Year 2 for 130,000 foreign
currencies (FC). The subsidiary pays for the inventory on January 31,Year 2.
Currency exchange rates for 1 foreign currency (FC) are as follows:
November 1 ,Year 1 P0.16 = 1 FC
December 31,Year 1 0.17 = 1
January 17,Year 2 0.18 = 1
January 31,Year 2 0.19 = 1
Average for Year 2 0.20 = 1

What amount does Benilan’s consolidated balance sheet report for this inventory at
December 31,Year 1?
P16,000 P18,000 P17,000 P19,000
General Feedback
P17,000

If the functional currency of the subsidiary is the local currency of a foreign


subsidiary, what exchange rates should be used to translate the items below to the
functional currency of the parent, assuming the foreign subsidiary is in a country
which has not experienced hyperinflation over three years?
Equipment Inventories Depreciation Expense – Equipment
Current Rate Current Rate Average Rate

Historical Current Rate Historical Rate


Rate

Current Rate Current Rate Historical Rate

Historical Rate Historical Rate Average Rate


General Feedback
Current Rate Current Rate Average Rate

If the functional currency is the local currency of a foreign subsidiary, what exchange
rates should be used to translate the items below, assuming the foreign subsidiary is in
a country which has not experienced hyperinflation over three years?
1) Equipment
2) Inventories
3) Depreciation Expense

1) Current Rate 2) Current Rate 3) Average Rate

1) Historical Rate 2) Current Rate 3) Historical Rate

1) Current Rate 2) Current Rate 3) Historical Rate

1) Historical Rate 2) Average Rate 3) Average Rate


General Feedback
1) Current Rate 2) Current Rate 3) Average Rate

A subsidiary of Salisbury, Inc. located in a foreign country whose functional currency


is the foreign currency (which is not currency of a hyperinflationary economy). The
subsidiary acquires inventory on credit on November 1,2017, for 100,000 foreign
currencies (FC) that is sold on January 17,2018 for 130,000 foreign currencies (FC).
The subsidiary pays for the inventory on January 31,2010. Currency exchange rates
for 1 foreign currency (FC) are as follows:
November 1 ,2017 P0.16 = 1
FC
December 31,2017 0.17 = 1
January 17,2018 0.18 = 1
January 31,2018 0.19 = 1
Average for 2018 0.20 = 1
What amount does Salisbury’s consolidated balance sheet report for this inventory at
December 31,2017?
P16,000 P18,000 P17,000 P19,000
General Feedback
P17,000

The following equity to an entity operating in a hyperinflationary economy:


Before After
PAS29 Restatement
Share capital 100 170
Revaluation 20 -
reserve
Retained earnings 30 -
150 270
What would be the balances on the revaluation reserve and retained earnings after the
restatement for PAS 29?
Revaluation reserve 0, retained earnings 100 Revaluation reserve 100, retained
earnings 0 Revaluation reserve 20, retained earnings 80 Revaluation reserve 70,
retained earnings 30
General Feedback
Revaluation reserve 0, retained earnings 100

UA Company was organized in January 1, 2012. Selected balances as of December 31,


2015 were as follows:
Land (revalued on December 31, 2014) 1,000,000
Factory building (constructed December 31, 2012) 500,000
Investment property (purchased on January 1, 800,000
2012)
Inventory 600,000
Note receivable (received January 1, 2015 200,000

The general price index had moved on December 31 of each year as follows:
December 31, 2012 – 140; December 31, 2013 – 190; December 31, 2014 – 240;
December 31, 2015 – 280

The restated amount for inventory is


646,154 1,200,000 700,000 790,588
General Feedback
Solution:
GPI, 12/31/2014 240
GPI, 12/31/2015 280
Total 520
Divide by: 2
Average GPI 260
Inventory, P600,000 x 280/260 = P646,154

UA Company was organized in January 1, 2012. Selected balances as of December 31,


2015 were as follows:
Land (revalued on December 31, 2014) 1,000,000
Factory building (constructed December 31, 2012) 500,000
Investment property (purchased on January 1, 800,000
2012)
Inventory 600,000
Note receivable (received January 1, 2015 200,000

The general price index had moved on December 31 of each year as follows:
December 31, 2012 – 140; December 31, 2013 – 190; December 31, 2014 – 240;
December 31, 2015 – 280

The fraction to be used in restating notes receivable


280/280 280/140 280/240 280/260
General Feedback
Solution:
Since notes receivable is a monetary item, it is not restated. Or if it is restated, the
fraction shall be equal to 1 based on the general price index as of the balance sheet date,
in this case, 280 as at December 31, 2015.

Royce Company operates in a hyperinflationary economy and provides the following


statement of financial position as of December 31, 2011
175,000
Cash
Inventory 1,350,000
Property, plant and equipment 450,000

Current liabilities 350,000


Non-current liabilities 250,000
Share capital (issuance date 200,000
2007)
Retained earnings 1,175,000

· The property, plant and equipment were purchased on December 31,


2009
· The non-current liabilities were loans raised on December 31, 2010
The general price index had moved each year as follows:
2007 100 2010 240
2008 130 2011 300
2009 150

The balance of property, plant and equipment after adjusting for hyperinflation
900,000 1,125,000 500,000 450,000
General Feedback
Solution:
Property, plant and equipment, P450,000 x 300/150 = P900,000

The following appear on the statement of financial position of AD Company


Cash in bank 1,000,000 Accounts payable 500,000
Accounts receivable 2,000,000 Accrued expenses 250,000
Advances to 100,000 Advances from 600,000
employee customers
Advances to 200,000 Unearned revenue 150,000
suppliers
Prepaid expenses 50,000 Estimated warranty 100,000
liability
Inventory 750,000 Bonds payable 1,500,000
Available-for-sale 250,000 Finance lease liability 2,000,000
securities
Patent 500,000 Deferred tax liability 200,000

In preparing financial statements in a hyperinflationary economy, the amount


classified as monetary liabilities is
1,500,000 2,250,000 4,250,000 4,850,000
General Feedback
Solution:
Accounts payable 500,000.00
Accrued expenses 250,000.00
Bonds payable 1,500,000.00
Finance lease liability 2,000,000.00
Total monetary liabilities 4,250,000.00

Property was purchased on December 31,2018 for 20 million baht. The general price
index in the country was 60.1 on that date. On December 31,2018, the general price index
had risen to 240.4. If the entity operates in a hyperinflationary economy, what be the
carrying amount in the financial statements of the property after restatement? 20
million baht 80 million baht 1,200.2 million baht 3,808 million baht
General Feedback
80 million baht

Royce Company operates in a hyperinflationary economy and provides the following


statement of financial position as of December 31, 2011
175,000
Cash
Inventory 1,350,000
Property, plant and equipment 450,000

Current liabilities 350,000


Non-current liabilities 250,000
Share capital (issuance date 200,000
2007)
Retained earnings 1,175,000

· The property, plant and equipment were purchased on December 31,


2009
· The non-current liabilities were loans raised on December 31, 2010
The general price index had moved each year as follows:
2007 100 2010 240
2008 130 2011 300
2009 150

The balance of the share capital after adjusting for hyperinflation


600,000 200,000 750,000 400,000
General Feedback
Solution:
Share capital, P200,000 x 300/100 = P600,000

In a business combination, an acquirer's interest in the fair value of the net assets
acquired exceeds the consideration transferred in the combination. Under
IFRS3 Business combinations, the acquirer should:

recognise the excess immediately in profit or loss

recognise the excess immediately in other comprehensive income

reassess the recognition and measurement of the net assets acquired and the consideration
transferred, then recognise any excess immediately in profit or loss
reassess the recognition and measurement of the net assets acquired and the consideration
transferred, then recognise any excess immediately in other comprehensive income
General Feedback
reassess the recognition and measurement of the net assets acquired and the consideration
transferred, then recognise any excess immediately in profit or loss

A business combination occurs when a company acquires an equity interest in another


entity and has

at least 20% ownership in the entity


more than 50% ownership in the entity

100% ownership in the entity


control over the entity, irrespective of the percentage owned
General Feedback
control over the entity, irrespective of the percentage owned

In a business combination, when the fair value exceeds the investment cost, which of the
following statements is correct?
A gain from a bargain purchase is recognized for the amount that the fair value of the
identifiable net assets acquired exceeds the acquisition price
the value is allocated first to reduce proportionately (according to market value) non-
current assets, then to non-monetary current assets, and any negative remainder is
classified as a deferred credit.

it is allocated first to reduce proportionately (according to market value) non-current


assets, and any negative remainder is classified as an extraordinary gain.

It is allocated first to reduce proportionately (according to market value) non-current,


depreciable assets to zero, and any negative remainder is classified as a deferred credit.
General Feedback
A gain from a bargain purchase is recognized for the amount that the fair value of the
identifiable net assets acquired exceeds the acquisition price

Kennedy Company is acquiring Ross Company in an acquisition. What date should be


used as the acquisition date for the transaction?
The date Kennedy signs the contract to purchase the business.

The date Kennedy obtains control of Ross.

The date that all contingencies related to the transaction are resolved.

The date Kennedy purchased more than 20% of the stock of Ross.
General Feedback

is incorrect – the date a contract is signed usually does not correspond with the date
control is acquired.is CORRECT - The acquisition date is the date the acquirer obtains
control of the acquire is incorrect – the acquisition may occur before all contingencies are
resolved.is incorrect – because control constitutes owning more than 50% of the shares of
stock outstanding

Which of the following is not one of those steps in accounting for an acquisition in
business combination?

Prepare pro forma financial statements prior to acquisition


Determine the acquisition date Identify the acquirer
Expense the costs and general expenses of the acquisition in the period of acquisition.
General Feedback
Prepare pro forma financial statements prior to acquisition

is CORRECT - Preparing pro forma financial statements prior to acquisition is not


required in the application of the acquisition methodis incorrect – identifying the acquirer
and determining the acquisition date are both steps in applying the acquisition method is
incorrect – identifying the acquirer and determining the acquisition date are both steps in
applying the acquisition methodis incorrect – direct costs of acquisitions and general
expenses related to an acquisition should be expensed in the period of acquisition

Lebow Corp. acquired control of Wilson Corp. by purchasing stock in steps. Which of the
following regarding this type of acquisition is true?
The cost of acquisition equals the amount paid for the previously held shares plus the fair
value of shares issued at the date of acquisition.

The previously held shares should be remeasured at fair value on the acquisition date, and
any gain on previously held shares should be included in other comprehensive income for
the period.

The previously held shares should be remeasured at fair value on the acquisition date and
the gain recognized in earnings of the period.

The acquisition cost includes only the newly issued shares measured at fair value on the
date of acquisition.
General Feedback
The previously held shares should be remeasured at fair value on the acquisition date and
the gain recognized in earnings of the period.

Answer (a) is incorrect - previously held shares are remeasured to fair value on the
acquisition date.Answer (b) is incorrect - any gain is recognized in earnings of the
period.Answer (c) is CORRECT – Any previously held shares should be remeasured at
fair value as of the date control is acquired, and the gain is recognized in earnings of the
period. If an unrealized gain was previously recognized in other comprehensive income,
the amount recognized in other comprehensive income should also be recognized as a
gain in the current periodAnswer (d) is incorrect - the previously issued shares must be
revalued at the acquisition date and included as part of the cost of the acquisition.

Which of the following is a reason why a company would expand through a


combination, rather than by building new facilities?

A combination might provide cost advantages.

A combination might provide fewer operating delays.

A combination might provide easier access to intangible assets.

All of the other choices are possible reasons that a company might choose a combination.
General Feedback
All of the other choices are possible reasons that a company might choose a combination.
Raphael Company paid 2,000,000 for the net assets of Paris Corporation and Paris
was then dissolved. Paris had no liabilities. The fair values of Paris’ assets were
2,500,000. Paris’s only non-current assets were land and equipment with fair values of
160,000 and 640,000, respectively. At what value will the equipment be recorded by
Raphael?

640,000

240,000

400,000

0
General Feedback
640,000

Goodwill arising from a business combination is

charged to Retained Earnings after the acquisition is completed.

amortized over 40 years or its useful life, whichever is longer.

amortized over 40 years or its useful life, whichever is shorter.

never amortized.
General Feedback
never amortized.

A business combination in which a new corporation is created and two or more


existing corporations are combined into the newly created corporation is called a

merger.
purchase transaction.

pooling-of-interests.

consolidation.
General Feedback
consolidation.

Which of the following situations would require the use of the acquisition method in a
business combination?

The acquisition of a group of assets.


The formation of a joint venture The purchase of more than 50% of a business

All of the above would require the use of the acquisition method.
General Feedback
The acquisition method applies only to acquisitions of a business. Acquisition of more than 50%
of the voting shares gives the acquirer presumptive control which would constitute an acquisition

On December 31, Year 2, Saxe Corporation was acquired by Poe Corporation. In the business
combination, Poe issued 200,000 shares of its 10 par common stock, with a market price of
18 a share, for all of Saxe’s common stock. The stockholders’ equity section of each
company’s balance sheet immediately before the combination was

Poe Saxe
Common stock 3,000,000 1,500,000
Additional paid-in capital 1,300,000 150,000
Retained earnings 2,500,000 850,000
6,800,000 2,500,000

In the December 31, Year 2 consolidated balance sheet, additional paid-in capital should be
reported at

950,000 1,300,000 1,450,000 2,900,000


General Feedback
In a business combination accounted for as an acquisition, the fair market value of the net assets
is used as the valuation basis for the combination. In this case, the net assets of the subsidiary
have an implied fair market value of $3,600,000 which is the value of the common stock issued
to Saxe’s shareholders (200,000 × $18). Since $3,600,000 is the basis for recording this
purchase, the common stock issued is recorded at $2,000,000 (200,000 shares × $10 par value
per share) and additional paid-in capital is recorded at $1,600,000 ($3,600,000 – $2,000,000).
Therefore, the additional paid-in capital should be reported at $2,900,000 ($1,300,000 +
$1,600,000).

In a business combination, when the fair value exceeds the investment cost, which of the
following statements is correct?

A gain from a bargain purchase is recognized for the amount that the fair value of the
identifiable net assets acquired exceeds the acquisition price.

the value is allocated first to reduce proportionately (according to market value) non-
current assets, then to non-A gain from a bargain purchase is recognized for the amount
that the fair value of the identifiable net assets acquired exceeds the acquisition
price.monetary current assets, and any negative remainder is classified as a deferred
credit.

it is allocated first to reduce proportionately (according to market value) non-current


assets, and any negative remainder is classified as an extraordinary gain.

It is allocated first to reduce proportionately (according to market value) non-current,


depreciable assets to zero, and any negative remainder is classified as a deferred credit.
General Feedback
A gain from a bargain purchase is recognized for the amount that the fair value of the
identifiable net assets acquired exceeds the acquisition price.

A business combination occurs when a company acquires an equity interest in another


entity and has

more than 50% ownership in the entity.

at least 20% ownership in the entity.

control over the entity


100% ownership in the entity.
General Feedback
Explanation: Business combination is a transaction or other event in which an acquirer
obtains control of one or more businesses [IFRS 3(2008) (Appendix A)], regardless of the
percentage of ownership.

On December 31, Year 2, Saxe Corporation was acquired by Poe Corporation. In the
business combination, Poe issued 200,000 shares of its 10 par common stock, with a
market price of 18 a share, for all of Saxe’s common stock. The stockholders’ equity
section of each company’s balance sheet immediately before the combination was

Poe Saxe
Common stock 3,000,000 1,500,000
Additional paid-in capital 1,300,000 150,000
Retained earnings 2,500,000 850,000
6,800,000 2,500,000

In the December 31, Year 2 consolidated balance sheet, additional paid-in capital
should be reported at

950,000

1,300,000

1,450,000

2,900,000
General Feedback
Suggested Solution
Poe Corporation
Fair Value of Shares issued in acquisition 3,600,000.00
Book value of shares issued (2,000,000.00)
Increase in APIC 1,600,000.00
APIC of acquirer before combination 1,300,000.00
Total APIC to be reported 2,900,000.00
On December 31, Year 1, Neal Co. issued 100,000 shares of its 10 par value common
stock in exchange for all of Frey Inc.’s outstanding stock. The fair value of Neal’s
common stock on December 31, Year 1, was 19 per share. The carrying amounts and
fair values of Frey’s assets and liabilities on December 31, Year 1, were as follows:

Carrying amount Fair value


Cash 240,000 240,000
Receivables 270,000 270,000
Inventory 435,,000 405,000
Property, plant, and 1,305,000 1,440,000
equipment
Liabilities (525,000) (525,000)
Net assets 1,725,000 1,830,000

What is the amount of goodwill resulting from the business combination?

175,000

105,000

70,000

0
General Feedback
Explanation: In a business combination accounted for as an acquisition, the fair market
value of the net assets is used as the valuation basis for the combination. In this case,
Frey’s assets have an implied fair market value of 1,900,000 which is the market value of
the common stock issue (100,000 shares × 19). The value assigned to goodwill is 70,000,
which is the value of the stock minus the fair value of Frey’s identifiable assets
(1,900,000 – 1,830,000).

TBB issued 120,000 shares of its 25par ordinary shares for all the net assets of HAF
Company on July 1, Year 2. TBB’s ordinary shares were selling at 30 per share at the
acquisition date. In addition a cash payment of 200,000 was made plus an agreed
deferred cash payment of 990,000 payable on July 1, Year 2. The market rate of
interest at the time is 10%.

TBB also agreed to pay additional cash consideration of 250,000 in the event TBB’s
net income falls below the current level within the next 2 years. TBB’s financial
officers were 99% sure the current level of income will at least be sustained during
the prescribed period.
The following out-of-pocket costs were paid in cash by TBB.

Legal and accounting fees paid to advisers 8,000


Broker's fees 4,000
Indirect acquisition costs 3,000
Costs to issue and register the shares 10,400
Total 25,400

Determine the cost of the investment for TBB

4,700,000

3,800,000

5,040,000

4,950,000
General Feedback
Suggested Solution
TBB
Consideration Transferred:
Shares (120,000sh * 30) 3,600,000.00
Cash 200,000.00
PV of Accounts Payable (deferred cash payment) 900,000.00
Contingent consideration -
Total Consideration Transferred = Cost of Investment 4,700,000.00

*note: the total consideration transferred is the cost of investment since there is no
existing equity ownership of acquirer in the acquiree entity. Should there be an existing
equity ownership, the fair value of such ownership (accounted as previously-held equity
instrument) shall be added to determine the total cost of investment

On April 1, Year 1, Dart Co. paid 620,000 for all the issued and outstanding common
stock of Wall Corp. The recorded assets and liabilities of Wall Corp. on April 1, Year
1, follow:
Cash 60,000
Inventory 180,000
Property and equipment (net of accumulated 320,000
depreciation of 220,000)
Goodwill 100,000
Liabilities (120,000)
Net assets 540,000

On April 1, Year 1, Wall’s inventory had a fair value of 150,000, and the property and
equipment (net) had a fair value of 380,000. What is the amount of goodwill resulting
from the business combination?

150,000

120,000

50,000

20,000
General Feedback
Suggested Solution
In an acquisition, the net assets of the acquired firm are recorded at their FV. The
excess of the cost of the investment over the FV of the net assets acquired is
allocated to goodwill. The cost of the investment is 620,000, and the FV of the net
assets acquired, excluding goodwill, is 470,000, as computed below.

FMV
Cash 60,000
Inventory (BV = 180,000) 150,000
Prop. and equip. (BV = 320,000) 380,000
Liabilities (120,000)
Total F 470,000

Therefore, the amount allocated to goodwill is 150,000 (620,000 – 470,000).

On January 1, Year 2, Carlito Company acquired 80% interests in Harries Company for
P2,000,000 cash. The stockholder’s equity of Harries at the time of acquisition is
P1,875,000. On January 1, Year 2, NCI is measured at its implied fair value. The excess
of cost over books value of interest acquired is allocated to the following assets:

Inventories P100, 000 (sold in Year 2)


Building P200, 000 (5- year remaining life)

During Year 2, Harries Company reported total comprehensive income of P500,000 and
paid dividend for P100,000.
What is the consolidated total comprehensive income attributable to parent on December
31, Year 2, if Carlito’s net income for Year 2 is P600,000?

876,000

888,000

808,000

948,000
General Feedback
808,000

On January 1, Year 2, Carlito Company acquired 80% interests in Harries Company for
P2,000,000 cash. The stockholder’s equity of Harries at the time of acquisition is
P1,875,000. On January 1, Year 2, NCI is measured at its implied fair value. The excess
of cost over books value of interest acquired is allocated to the following assets:

Inventories P100, 000 (sold in Year 2)


Building P200, 000 (5- year remaining life)

During Year 2, Harries Company reported total comprehensive income of P500,000 and
paid dividend for P100,000.

What was the fair value of NCI on January 1, Year 2?

P500,000

P375,000

P525,000

P400,000
General Feedback
P500,000
On January 1, Year 2, Carlito Company acquired 80% interests in Harries Company for
P2,000,000 cash. The stockholder’s equity of Harries at the time of acquisition is
P1,875,000. On January 1, Year 2, NCI is measured at its implied fair value. The excess
of cost over books value of interest acquired is allocated to the following assets:

Inventories P100, 000 (sold in Year 2)


Building P200, 000 (5- year remaining life)

During Year 2, Harries Company reported total comprehensive income of P500,000 and
paid dividend for P100,000.

How much goodwill (gain on acquisition) is reported in the consolidated statement of


financial position on 1/1/Year 2?

P325,000

P200,000

P(325,000)

P(375,000)
General Feedback
P325,000

Sub Company sells all its output at 20 percent above cost to Par Corporation. Par
purchases all its inventory from Sub. The incomes reported by the companies over the
past three years are as follows:

Year Sub Company’s Net Income Par Corporation’s Operating Income


Year 1 150,000 225,000
Year 2 135,000 360,000
Year 3 240,000 450,000

Sub Company sold inventory for P300,000, P262,500 and P337,500 in the years Year 1,
Year 2, and Year 3 respectively. Par Company reported ending inventory of P105,000,
P157,500 and P180,000 for Year 1, Year 2, and Year 3 respectively. Par acquired 70
percent of the ownership of Sub on January 1, Year 1, at underlying book value. The fair
value of the noncontrolling interest at the date of acquisition was equal to 30 percent of
the book value of Sub Company.
What will be the income assigned to controlling interest for Year 2?

P448,375

P495,000

P486,250

P615,375
General Feedback
Suggested Solution
Profit - Par, Year 2 360,000.00
Share in Profit - Sub, Year 2 94,500.00
Share in RGP in beginning Inventory (105,000/120%*20%*70%) 12,250.00
Share in DGP in Ending Inventory (157,500/120%*20%*70%) (18,375.00)
Profit - Equity Holders of Parent 448,375.00

On January 1, Year 1, Wilhelm Corporation acquired 90 percent of Kaiser Company's


voting stock, at underlying book value. The fair value of the noncontrolling interest was
equal to 10 percent of the book value of Kaiser at that date. Wilhelm uses the equity
method in accounting for its ownership of Kaiser. On December 31, Year 2, the trial
balances of the two companies are as follows:
Wilhelm Corporation Kaiser Company
Debit Credit Debit Credit
Current Assets 200,000 140,000
Depreciable Assets 350,000 250,000
Investment in Kaiser Company 162,000
Stock
Depreciation Expense 27,000 10,000
Other Expenses 95,000 60,000
Dividends Declared 20,000 10,000
Accumulated Depreciation 118,000 80,000
Current Liabilities 100,000 80,000
Long-Term Debt 100,000 50,000
Common Stock 100,000 50,000
Retained Earnings 150,000 100,000
Sales 250,000 110,000
Income from Subsidiary 36,000

Based on the preceding information, what amount would be reported as retained earnings
in the consolidated balance sheet prepared at December 31, Year 2?
314,000

294,000

150,000

424,000
General Feedback
Suggested Solution
Wilhelm Corporation
Retained Earnings - Parent 150,000.00
Profit - Parent
Sales 250,000.00
Depreciation Expense (27,000.00)
Other expenses (95,000.00) 128,000.00
Dividends Declared - Parent (20,000.00)
Retained Earnings - Parent, Ending 258,000.00
Parent's share in Subsidiary Profit - 2009 36,000.00
Retained Earnings - Equity Holders of Parent 294,000.00

Pact acquired 80% of the equity shares of Sact on 1 July Year 1, paying P3.00
for each share acquired. This represented a premium of 20% over the market price of
Sact’s shares at that date. Sact’s shareholders’ funds (equity) as at 31 March Year 2 were:

Equity shares of P1 each 100,000


Retained earnings at 1 April Year 1 80,000
Profit for the year ended 31 March 40,000 120,000
Year 2
220,000

The only fair value adjustment required to Sact’s net assets on consolidation was a
P20,000 increase in the value of its land. Pact’s policy is to value non-controlling
interests at fair value at the date of acquisition. For this purpose the market price of
Sact’s shares at that date can be deemed to be representative of the fair value of the
shares held by the non-controlling interest.

What would be the carrying amount of the non-controlling interest ofSact in the
consolidated statement of financial position of Pact as at 31 March Year 2?
P58,000

P56,000

P54,000

P50,000
General Feedback
Suggested Solution
Pact and Sact
Equity shares, March 1, Year 2 (# of shares) 100,000.00
Multiply by: NCI percentage 20%
NCI held shares 20,000.00
Multiply by: FV/Share (P3 / 120%) 2.50
NCI - Net asset, at acquisition* 50,000.00
Share in profit (40,000 * 9/12 * 20%) 6,000.00
Share in dividends -
NCI - Net asset, March 31, Year 2 56,000.00

A subsidiary made sales of inventory to its parent at a profit this year. The parent, in turn,
sold all but 20 percent of the inventory to unaffiliated companies, recognizing a profit.
The amount that should be reported as cost of goods sold in the consolidated income
statement prepared for the year should be:

the amount reported as intercompany sales by the subsidiary.

the amount reported as intercompany sales by the subsidiary minus unrealized profit in
the ending inventory of the parent.

the amount reported as cost of goods sold by the parent minus unrealized profit in the
ending inventory of the parent.

the amount reported as cost of goods sold by the parent.


General Feedback
the amount reported as intercompany sales by the subsidiary minus unrealized profit in
the ending inventory of the parent.
Novy Corporation purchased at book value 70 percent of the ownership of Meiji
Corporation and 90 percent of the ownership of Cecille Corporation in Year 1. There are
frequent intercompany transfers among the companies. Activity relevant to Year 4 is
presented below.
Production Transfer Unsold at
Year Producer Cost Buyer Price End of Year Year Sold
Year 3 Meiji Corp. 24,000 Novy Corp. 30,000 10,000 Year 4
Year 3 Cecille Corp. 60,000 Meiji Corp. 72,000 18,000 Year 4
Year 4 Novy Corp. 15,000 Meiji Corp. 35,000 7,000 Year 5
Cecille
Year 4 Meiji Corp. 63,000 Corp. 72,000 12,000 Year 5
Year 4 Cecille Corp. 27,000 Novy Corp. 45,000 15,000 Year 5

For the year ended December 31, Year 4, Novy Corporation reported P80,000 of income
from its separate operations (excluding income from intercorporate investments). Meiji
Corp. reported net income of P37,500, and Cecille Corporation reported net income of
P20,000.

Compute the amount reported as consolidated net income for Year 4.

P 117,900

P 116,750

P 142,50

P 96,750
General Feedback
Suggested Solution
Novy Corporation EHP Consolidated
Profit - Parent (Novy) - separate operations 80,000.00 80,000.00
Profit - Subsidiary (Meiji) 26,250.00 37,500.00
Profit - Subsidiary (Cecille) 18,000.00 20,000.00
Intercompany Sales
RGP: Meiji - Novy [10,000*20%] 1,400.00 2,000.00
RGP: Cecille - Meiji [18,000/120%*20%] 2,700.00 3,000.00
UGP: Novy - Meiji [7,000*57.14%] (3,999.80) (3,999.80)
UGP: Meiji - Cecille [12,000*12.5%] (1,050.00) (1,500.00)
UGP: Cecille - Novy [15,000*40%] (5,400.00) (6,000.00)
Total Profit 117,900.20 131,000.20
*the amount requested is "consolidated Profit", but amount included in choices is
"consolidated profit attributable to parent"
**kindly observe the computation for inter-subsidiary sale
BaduyCorp. owns 80 percent of the stock of Hiphop Company. At the end of Year 2,
Baduy Corp. and Hiphop Company reported the following partial operating results and
inventory balances:
Baduy Corp. Hiphop Co.
Total sales 658,000 510,000
Sales to Hiphop Co. 140,000
Sales to Baduy Corp. 240,000
Profit 20,000
Operating Profit (excluding income from Hiphop 70,000
Co.)
Inventory, December 31, Year 2:
Purchases from Hiphop Co. 48,000
Purchases from Baduy Corp. 42,000

Baduy Corporation regularly prices its products at cost plus a 40 percent mark-up for
profit. Hiphop Company prices its sales at cost plus a 20 percent mark-up. The total sales
reported by Baduy and Hiphop include both intercompany sales and sales to nonaffiliates.

The consolidated cost of sales for Year 2 must be:

790,000

770,000

535,000

496,333
General Feedback
Suggested Solution
baduy Corp
Cost of goods sold - Parent (658,000/140%) 470,000
Cost of goods sold - Subsidiary (510,000/120%) 425,000
Intercompany Sale (140,000+240,000) (380,000)
Unrealized gross profit on ending inventory
[(48,000/120%)*20%] (6,667)
[(42,000/140%)*40%] (12,000)
Consolidated Cost of Goods Sold 496,333
On January 1, Year 1 SST Company purchased a computer with an expected life of 5
years. On January 1, Year 3 SST Company sold the computer to PMN corporation and
recorded the following entry:
Cash P39, 000
Accumulated Depreciation 16, 000
Computer Equipment 40, 000
Gain on sale of equipment 15, 000

PMN Corporation holds 60% of the voting shares of SST Company. SST Company and
PMN Corporation reported income from its own operations of P45, 000 and P85, 000 for
Year 3 respectively. There is no change in the estimated life of the equipment as a result
of intercompany sale.

How much is the income attributable to the Non-Controlling Interest for Year 3?

12,000

14,000

18,000

21,000
General Feedback
PMN Corporation and SST Company
NCI
Profit - Parent (own operation)
Profit - Subsidiary 18,000
Intercompany sale of PPE
Eliminate Gain (6,000)
Amortization of gain 2,000
Total 14,000

Penny Company owns an 80% controlling interest in the Sandy’s Company. Sandy
regularly sells merchandise to Penny, which then sold to outside parties. The gross profit
on all such sales is 40%. On January 1, Year 2, Penny sold land and a building to Sandy.
The value of the parcel is 20% to land and 80% to structures. Pertinent data for the
companies is summarized in the next page.

Penny Sandy
Internally generated net income, Year 2 520,000 250,000
Internally generated net income, Year 3 340,000 235,000
Intercompany merchandise sales, Year 2 100,000
Intercompany merchandise sales, Year 3 120,000
Intercompany inventory, December 31, Year 2 15,000
Intercompany inventory, December 31, Year 3 20,000
Cost of real estate sold on January 1, Year 2 600,000
Sales price of real estate on January 1, Year 2 800,000
Depreciable life of building 20 yrs

For Year 2, what is the consolidated comprehensive income attributable to controlling


interest?

523,200

525,000

625,000

532,500
General Feedback
523,200

KEV Corporation’s stockholder’s equity at December 31, Year 2 included the following:
8% Preferred stock, 10 par value P 3,500,000
Common stock, no par 20,000,000
Additional paid-in capital 6,500,000
Retained earnings 8,000,000
P 38,000,000

ROF Corporation purchased a 30% interest in KEV’s common stock from other shareholders
on January 1, Year 3 for 11,600,000. What was the book value of ROF’s investment in KEV?

10,3500,000

11,400,000
11,400,000

10,800,000
General Feedback
Total stockholders’ equity 38,000,000
Less: preferred equity 3,500,000
Equals: common equity 34,500,000
x ROF’s percentage 30%
Book value of Rof 10,350,000
investment

On January 1, Year 1, Polk Corp. and Strass Corp. had condensed balance sheets as follows:
Polk Strass
Current assets $ 70,000 $20,000
Noncurrent assets 90,000 40,000
Total assets $160,000 $60,000
Current liabilities 30,000 10,000
Long-term debt 50,000 --
Stockholders’ equity 80,000 50,000
Total liabilities and stockholders’ equity 160,000 60,000

On January 2, Year 1, Polk borrowed $60,000 and used the proceeds to purchase 90% of the
outstanding common shares of Strass. This debt is payable in ten equal annual
principal payments, plus interest, beginning December 30, Year 1. The excess cost of the
investment over Strass’ book value of acquired net assets should be allocated 60% to inventory
and 40% to goodwill. On January 1, Year 1, the fair
value of Polk shares held by noncontrolling parties was $10,000.

On Polk’s January 2, Year 1 consolidated balance sheet


Noncurrent assets should be
$130,000 $136,000 $138,000 $140,000
General Feedback
The acquisition method requires the assets of the acquired firm to be recorded at their fair values.
The fair value of the net assets of the acquiree is determined by adding the acquirer’s cost to the
fair value of non-controlling interest. In this case the fair value would be equal to $70,000
($60,000 + $10,000). The excess of the fair value over book value is allocated 60% to inventory
and 40% to goodwill. Goodwill would be recorded at $8,000 ($20,000 × 40%). Therefore,
noncurrent assets should be reported at $138,000 as calculated below.

Which of the following is not one of those steps in accounting for an acquisition in
business combination?

Prepare pro forma financial statements prior to acquisition


Determine the acquisition date Identify the acquirer
Expense the costs and general expenses of the acquisition in the period of acquisition.
General Feedback
Prepare pro forma financial statements prior to acquisition

is CORRECT - Preparing pro forma financial statements prior to acquisition is not


required in the application of the acquisition methodis incorrect – identifying the acquirer
and determining the acquisition date are both steps in applying the acquisition method is
incorrect – identifying the acquirer and determining the acquisition date are both steps in
applying the acquisition methodis incorrect – direct costs of acquisitions and general
expenses related to an acquisition should be expensed in the period of acquisition

In a business combination, when the fair value exceeds the investment cost, which of the
following statements is correct?
A gain from a bargain purchase is recognized for the amount that the fair value of the
identifiable net assets acquired exceeds the acquisition price
the value is allocated first to reduce proportionately (according to market value) non-
current assets, then to non-monetary current assets, and any negative remainder is
classified as a deferred credit.

it is allocated first to reduce proportionately (according to market value) non-current


assets, and any negative remainder is classified as an extraordinary gain.

It is allocated first to reduce proportionately (according to market value) non-current,


depreciable assets to zero, and any negative remainder is classified as a deferred credit.
General Feedback
A gain from a bargain purchase is recognized for the amount that the fair value of the
identifiable net assets acquired exceeds the acquisition price

On December 31, Year 2, Saxe Corporation was acquired by Poe Corporation. In the
business combination, Poe issued 200,000 shares of its 10 par common stock, with a
market price of 18 a share, for all of Saxe’s common stock. The stockholders’ equity
section of each company’s balance sheet immediately before the combination was

Poe Saxe
Common stock 3,000,000 1,500,000
Additional paid-in capital 1,300,000 150,000
Retained earnings 2,500,000 850,000
6,800,000 2,500,000

In the December 31, Year 2 consolidated balance sheet, additional paid-in capital
should be reported at
950,000 1,300,000 1,450,000 2,900,000
General Feedback
In a business combination accounted for as an acquisition, the fair market value of the net
assets is used as the valuation basis for the combination. In this case, the net assets of the
subsidiary have an implied fair market value of $3,600,000 which is the value of the
common stock issued to Saxe’s shareholders (200,000 × $18). Since $3,600,000 is the
basis for recording this purchase, the common stock issued is recorded at $2,000,000
(200,000 shares × $10 par value per share) and additional paid-in capital is recorded at
$1,600,000 ($3,600,000 – $2,000,000). Therefore, the additional paid-in capital should be
reported at $2,900,000 ($1,300,000 + $1,600,000).

KEV Corporation’s stockholder’s equity at December 31, Year 2 included the


following:
8% Preferred stock, 10 par value P 3,500,000
Common stock, no par 20,000,000
Additional paid-in capital 6,500,000
Retained earnings 8,000,000
P 38,000,000

ROF Corporation purchased a 30% interest in KEV’s common stock from other
shareholders on January 1, Year 3 for 11,600,000. What was the book value of ROF’s
investment in KEV?

10,3500,000

11,400,000
11,400,000

10,800,000
General Feedback
Total stockholders’ equity 38,000,000
Less: preferred equity 3,500,000
Equals: common equity 34,500,000
x ROF’s percentage 30%
Book value of Rof 10,350,000
investment

On November 30, Year 1, Parlor, Inc. purchased for cash at $15 per share all 250,000
shares of the outstanding common stock of Shaw Co. At November 30, Year 1, Shaw’s
balance sheet showed a carrying amount of net assets of $3,000,000. At that date, the fair
value of Shaw’s property, plant and equipment exceeded its carrying amount by
$400,000. In its November 30, Year 1 consolidated balance sheet, what amount should
Parlor report as goodwill? $750,000 $400,000 $350,000
$0
General Feedback
In an acquisition, the net assets of the acquired firm are recorded at their FV. The excess
of the cost of theinvestment over the FV of the net assets acquired is allocated to
goodwill. The cost of this investment is $3,750,000 (250,000 shares × $15), and the FV of
the net assets acquired,excluding goodwillis $3,400,000 ($3,000,000 + $400,000).
Therefore, the amount allocated to goodwill is $350,000 ($3,750,000 – $3,400,000).

On January 1, Year 1, Polk Corp. and Strass Corp. had condensed balance sheets as
follows:
Polk Strass
Current assets $ 70,000 $20,000
Noncurrent assets 90,000 40,000
Total assets $160,000 $60,000
Current liabilities 30,000 10,000
Long-term debt 50,000 --
Stockholders’ equity 80,000 50,000
Total liabilities and stockholders’ equity 160,000 60,000

On January 2, Year 1, Polk borrowed $60,000 and used the proceeds to purchase 90% of
the outstanding common shares of Strass. This debt is payable in ten equal annual
principal payments, plus interest, beginning December 30, Year 1. The excess cost of the
investment over Strass’ book value of acquired net assets should be allocated 60% to
inventory and 40% to goodwill. On January 1, Year 1, the fair
value of Polk shares held by noncontrolling parties was $10,000.

On Polk’s January 2, Year 1 consolidated balance sheet


Noncurrent assets should be

$130,000 $136,000 $138,000 $140,000


General Feedback
The acquisition method requires the assets of the acquired firm to be recorded at their fair
values. The fair value of the net assets of the acquiree is determined by adding the
acquirer’s cost to the fair value of non-controlling interest. In this case the fair value
would be equal to $70,000 ($60,000 + $10,000). The excess of the fair value over book
value is allocated 60% to inventory and 40% to goodwill. Goodwill would be recorded at
$8,000 ($20,000 × 40%). Therefore, noncurrent assets should be reported at $138,000 as
calculated below.

On April 1, year 1, Parson Corp. purchased 80% of the outstanding stock of Sloan Corp.
for 700,000 cash. Parson determined that the fair value of the net identifiable assets was
800,000 on the date of acquisition. The fair value of Sloan’s stock at date of acquisition
was 18 per share. Sloan had a total of 50,000 shares of stock issued and outstanding prior
to the acquisition. What is the amount of goodwill that should be recorded by Parson at
date of acquisition?
0
60,000 80,000 120,000
General Feedback
80,000

When does the measurement period end for a business combination in which there was
incomplete accounting information on the date of acquisition? When the acquirer
receives the information or one year from the acquisition date, whichever occurs earlier.
On the final date when all contingencies are resolved. Thirty days from the date of
acquisition. At the end of the reporting period in the year of acquisition.
General Feedback
When the acquirer receives the information or one year from the acquisition date,
whichever occurs earlier.
On June 30, 2018, Wyler Corporation acquires Boston Corporation in a transaction
properly accounted for as a business acquisition. At the time of the acquisition, some of
the information for valuing assets was incomplete. How should Corporation Wyler,
account for the incomplete information in preparing its financial statements immediately
after the acquisition? Do not record the uncertain items until complete information is
available. Record a contra account to the investment account for the amounts
involved. Record the uncertain items at the book value of the acquiree. Record the
uncertain items at a provisional amount measured at the date of acquisition.
General Feedback
Record the uncertain items at a provisional amount measured at the date of acquisition.

Which of the following is false relating to reverse acquisition? Reverse acquisition


may transpire only if the consideration transferred includes shares of stocks of the
acquiring entity. The acquirer in a reverse acquisition is designated as the legal
acquiree. Non-controlling interest shareholders in a reverse acquisition are
shareholders of the accounting acquirer entity Non-controlling interest in a reverse
acquisition may be valued at the fair value of the shares held by the non-controlling
interest shareholders
General Feedback
Non-controlling interest in a reverse acquisition may be valued at the fair value of the
shares held by the non-controlling interest shareholders

Which of the following is true pertaining to reverse acquisition? Legal acquirer is the
accounting acquirer Legal acquirer is the accounting acquiree Non-controlling
interest are shareholders of legal acquirer Non-controlling interest are shareholders of
accounting acquiree
General Feedback
Legal acquirer is the accounting acquiree

On January 1, 2017, Palm, Inc. purchased 80% of the stock of Stone Corp. for 4,000,000
cash. Prior to the acquisition, Stone had 100,000 shares of stock outstanding. On the date
of acquisition, Stone’s stock had a fair value of 52 per share. During the year Stone
reported 280,000 in net income and paid dividends of 50,000. What is the balance in the
noncontrolling interest account on Palm’s balance sheet on December 31, 2017?
1,000,000 1,040,000 1,086,000 1,096,000
General Feedback
1,086,000
On January 1, Year 2, Carlito Company acquired 80% interests in Harries Company for
P2,000,000 cash. The stockholder’s equity of Harries at the time of acquisition is
P1,875,000. On January 1, Year 2, NCI is measured at its implied fair value. The excess
of cost over books value of interest acquired is allocated to the following assets:

Inventories P100, 000 (sold in Year 2)


Building P200, 000 (5- year remaining
life)

During Year 2, Harries Company reported total comprehensive income of P500,000 and
paid dividend for P100,000.
What was the fair value of NCI on January 1, Year 2?

P500,000 P375,000 P525,000 P400,000


General Feedback
P500,000

A subsidiary was acquired for cash in a business combination on January 1, 2017. The
consideration given exceeded the fair value of identifiable net assets. The acquired
company owned equipment with a market value in excess of the carrying amount as of the
date of combination. A consolidated balance sheet prepared on December 31, 2017,
would Report the unamortized portion of the excess of the market value over the
carrying amount of the equipment as part of goodwill. Report the unamortized portion
of the excess of the market value over the carrying amount of the equipment as part of
plant and equipment. Report the excess of the market value over the carrying amount
of the equipment as part of plant and equipment. Not report the excess of the market
value over the carrying amount of the equipment because it would be expensed in the
year of the acquisition
General Feedback
Report the unamortized portion of the excess of the market value over the carrying
amount of the equipment as part of plant and equipment.

Parker Corp. owns 80% of Smith Inc.’s common stock. During 2017, Parker sold
Smith 250,000 of inventory on the same terms as sales made to third parties. Smith
sold all of the inventory purchased from Parker in 2017. The following information
pertains to Smith and Parker’s sales for 2017:
Parker Smith
Sales 1,000,000 700,000
Cost of sales 400,000 350,000
600,000 350,000
What amount should Parker report as cost of sales in it s 2017 consolidated income
statement?
750,000 680,000 500,000 430,000
General Feedback
500,000

On January 1, Year 1, Wilhelm Corporation acquired 90 percent of Kaiser Company's


voting stock, at underlying book value. The fair value of the noncontrolling interest was
equal to 10 percent of the book value of Kaiser at that date. Wilhelm uses the equity
method in accounting for its ownership of Kaiser. On December 31, Year 2, the trial
balances of the two companies are as follows:
Wilhelm Corporation Kaiser Company
Debit Credit Debit Credit
Current Assets 200,000 140,000
Depreciable Assets 350,000 250,000
Investment in Kaiser 162,000
Company Stock
Depreciation Expense 27,000 10,000
Other Expenses 95,000 60,000
Dividends Declared 20,000 10,000
Accumulated Depreciation 118,000 80,000
Current Liabilities 100,000 80,000
Long-Term Debt 100,000 50,000
Common Stock 100,000 50,000
Retained Earnings 150,000 100,000
Sales 250,000 110,000
Income from Subsidiary 36,000
Based on the preceding information, what amount would be reported as retained earnings
in the consolidated balance sheet prepared at December 31, Year 2?
314,000 294,000 150,000 424,000
General Feedback
Wilhelm Corporation
Retained Earnings - Parent 150,000.00
Profit - Parent
Sales 250,000.00
Depreciation Expense (27,000.00)
Other expenses (95,000.00) 128,000.00
Dividends Declared - Parent (20,000.00)
Retained Earnings - Parent, Ending 258,000.00
Parent's share in Subsidiary Profit -
2009 36,000.00
Retained Earnings - Equity Holders
of Parent 294,000.00
Wilmslow acquired 80% of the equity shares of Zeta on 1 April Year 1 when Zeta’s
retained earnings were P200,000. During the year ended 31 March Year 2, Zeta
purchased goods from Wilmslow totalling P320,000. At 31 March Year 2, one quarter
of these goods were still in the inventory of Zeta. Wilmslow applies a mark-up on cost
of 25% to all of its sales. At 31 March Year 2, the retained earnings of Wilmslow and
Zeta were P450,000 and P340,000 respectively.

What would be the amount of retained earnings in Wilmslow’s consolidated statement


of financial position as at31 March Year 2?
P706,000 P546,000 P542,000 P498,0
General Feedback
Wilmslow
Retained Earnings, end - Parent - separate FS 450,000.00
Unrealized gross profit on Downstream sales
[(320,000/4)/125%)*25%] (16,000.00)
Share in adjusted profit of
Subsidiary
Subsidiary reported profit [(340,000-
200,000)*80%] 112,000.00
Consolidated Retained Earnings 546,000.00

A subsidiary made sales of inventory to its parent at a profit this year. The parent, in turn,
sold all but 20 percent of the inventory to unaffiliated companies, recognizing a profit.
The amount that should be reported as cost of goods sold in the consolidated income
statement prepared for the year should be: the amount reported as intercompany sales
by the subsidiary the amount reported as intercompany sales by the subsidiary minus
unrealized profit in the ending inventory of the parent the amount reported as cost of
goods sold by the parent minus unrealized profit in the ending inventory of the parent
the amount reported as cost of goods sold by the parent
General Feedback
the amount reported as intercompany sales by the subsidiary minus unrealized profit in
the ending inventory of the parent

Planet Company acquired a 70% interest in the Star Company in 2016. For the year ended
December 31, 2017, Star reported net income of 80,000. During 2017, Planet sold
merchandise to Star for 10,000 at a profit of 2,000. The merchandise remained in Star’s
inventory at the end of 2017. For consolidation purposes what is the noncontrolling
interest’s share of Star’s net income for 2017? 23,400 24,000 24,600 26,000
General Feedback
24,000

Pat Corp. owns 80% for Sir. Inc. common stock. During Year 2, Pat sold Sir P250,000
of inventory on the same terms as sale made to third parties. Sir sold all of the
inventory purchased from Pat in Year 2. The following information pertains for Sir
and Pat’s sales for Year 2.

Pat Sir
P1,000,00
Sales P700,000
0
Cost of
400,000 350,000
sales
Gross profit P600,000 P350,000

What amount should Pat report as cost of sales in its Year 2 consolidated statement of
comprehensive income? P750,000 P680,000 P500,000 P430,000
General Feedback
P500,000

Port, Inc. owns 100% of Salem, Inc. On January 1, 2018, Port sold Salem delivery
equipment at a gain. Port had owned the equipment for two years and used a five-year
straight-line depreciation rate with no residual value. Salem is using a three-year straight-
line depreciation rate with no residual value for the equipment. In the consolidated
income statement, Salem’s recorded depreciation expense on the equipment for 2018 will
be decreased by

20% of the gain on sale. 33 1/3% of the gain on sale. 50% of the gain on sale.
100% of the gain on sale
General Feedback
33 1/3% of the gain on sale.

On January 1, Year 1 SST Company purchased a computer with an expected life of 5


years. On January 1, Year 3 SST Company sold the computer to PMN corporation and
recorded the following entry:
Cash P39, 000
Accumulated
16, 000
Depreciation
Computer Equipment 40, 000
Gain on sale of
15, 000
equipment

PMN Corporation holds 60% of the voting shares of SST Company. SST Company and
PMN Corporation reported income from its own operations of P45, 000 and P85, 000 for
Year 3 respectively. There is no change in the estimated life of the equipment as a result
of intercompany sale.
What is the consolidated total comprehensive income attributable to parent for Year 3?

P103, 000 P106, 000 - answer


P112, 000 P130, 000
General Feedback
PMN Corporation and SST Company
EHP NCI Consolidated
Profit - Parent (own operation) 85,000 85,000
Profit - Subsidiary 27,000 18,000 45,000
Intercompany sale of PPE
Eliminate Gain (9,000) (6,000) (15,000)
Amortization of
gain 3,000 2,000 5,000
Total 106,000 14,000 120,000
#12 #13

Penny Company owns an 80% controlling interest in the Sandy’s Company. Sandy
regularly sells merchandise to Penny, which then sold to outside parties. The gross profit
on all such sales is 40%. On January 1, Year 2, Penny sold land and a building to Sandy.
The value of the parcel is 20% to land and 80% to structures. Pertinent data for the
companies is summarized in the next page.

Penny Sandy
Internally generated net income, Year 2 520,000 250,000
Internally generated net income, Year 3 340,000 235,000
Intercompany merchandise sales, Year 2 100,000
Intercompany merchandise sales, Year 3 120,000
Intercompany inventory, December 31,
Year 2 15,000
Intercompany inventory, December 31,
Year 3 20,000
Cost of real estate sold on January 1, Year
2 600,000
Sales price of real estate on January 1, Year
2 800,000
Depreciable life of building 20 yrs
For Year 3, what is the consolidated comprehensive income attributable to controlling
interest?
534,400 543,000 453,400 543,400
General Feedback
534,400

BigBang Company owns an 80% controlling interest in Sheldon Company. Sheldon


regularly sells merchandise to BigBang, which then sells to outside parties. The gross
profit on all such sales is 40%. On January 1, Year 1, BigBang sold land and a building to
Sheldon. The value of the parcel is 20% to land and 80% to structures. Pertinent data for
the companies is summarized below.
BigBang Sheldon
Internally generated net income, Year 2 340, 000 235, 000
Internally generated net income, Year 1 P520, 000 P250, 000
Intercompany merchandise sales, Year 2 120, 000
Intercompany merchandise sales, Year 1 100, 000
Intercompany inventory, December 31, Year 2 20, 000
Intercompany inventory, December 31, Year 1 15, 000
Cost of real estate sold on January 1, Year 1 600, 000
Sales price of real estate on January 1, Year 1 800, 000
Depreciable life of building 20 years.

For Year 2, what is the consolidated comprehensive income attributable to controlling


interest?

453, 400

534, 400

543, 000

543, 400
General Feedback
Suggested Solution
BigBang Company
EHP
Profit - Parent (own operation) 340,000
Profit - Subsidiary 188,000 (235,000*80%)
Intercompany sale - Inventory (upstream)
Unrealized gross profit (6,400) (20,000*40%*80%)
Realized gross profit 4,800 (15,000*40%*80%)
Intercompany sale - PPE (downstream)
PPE - amortization of
gain 8,000 [((800,000-600,000)*80%)/20]
Profit - EHP 534,400

On July 1, Year 1, Eliza, Rochie and Jessa formed a joint arrangement for the sale of
merchandise. Eliza was designated as the managing joint operator. Profits or losses
are to be divided as follows: Eliza, 50%; Rochie, 25%; and Jessa, 25%. On October 1,
Year 1, though the joint operation is still uncompleted, the participants agreed to
recognize profit or loss on the venture to date. The cost of inventory on hand is
determined at P25,000. The investment in Joint Operation account has a debit balance
of P15,000 before distribution of profit and loss. No separate set of books is
maintained for the joint operation and the participants record in their individual books
all venture transactions.

The joint operation profit (loss) on October 1, Year 1 is:

10,000

25,000

(15,000)

None
General Feedback
Suggested Solution
Eliza, Rochie, and Jessa
Operation before P/L 15,000.00
Unsold merchandise 25,000.00
Profit 10,000.00

K and L[1] join in a venture for the sale of certain merchandise. The participants agree to
the following:
· K shall be allowed a commission of 10% on his net purchase.
· The participants shall be allowed commissions of 25% on their respective sales.
· K and L shall divide the profit or loss 60% and 40%, respectively.
Joint arrangement transactions follows:
Dec.
1 K makes cash purchase of P57,000
3 L pays venture expenses of P9,000.
Sales are as follows: K P48,000; L P36,000. The participants keep
5 their own cash receipts.
6 K returns unsold merchandise and receives P15,000 cash.
15 The participants make cash settlement.

In the distribution of the net profit of the venture, what are the shares of K and L,
respectively?

4,260 3,230

4,680 3,120

4,820 3,430

4,840 4,230
General Feedback
Suggested Solution JA K L
Sales 84,000 (48,000) (36,000)
Purchases (57,000) 57,000
Expenses (9,000) 9,000
Purchase returns 15,000 (15,000)
Profit 33,000
Allocation of profit
Commission - purchases (4,200) 4,200
Commission - sales (21,000) 12,000 9,000
Balance (7,800) 4,680 3,120 #20
Settlement 33,000 14,880 (14,880) #21

Three joint operators are involved in a joint operation that manufactures ships chandlery.
At the beginning of the year the joint operation held P50,000 in cash. During the year the
joint operation incurred the following expenses: Wages paid P20,000, Overheads accrued
P10,000. Additionally, creditors amounting to P40,000 were paid and the joint operators
contributed P15,000 cash each to the joint operation. The balance of cash held by the
joint operation at the end of the year is:
P5,000

P25,000

P35,000

P75,000
General Feedback
Suggested Solution
Cash, beginning 50,000
Joint operators' additional investment 45,000
Disbursments
Wages (20,000)
Payment of accounts (40,000)
Cash, ending 35,000

On January 2, Year 1, Abnoy Company and Sibuyas Company formed the DILAWAN
Company, a merchandising joint venture intended to prevent any political identity to
sit in the government without their approval. Each invested P200,000 for a 50%
interest in the joint venture with the agreement that the managing group is awarded
first to Abnoy. The venture’s operation went smoothly as nobody noticed their
scheme.
The condensed financial statements for Abnoy Company, Sibuyas Company and for
the joint venture, Dilawan Company are presented below:
Dilawan Company
Abnoy Co. Sibuyas Co. (a joint venture)
Profit or Loss:
Sales P3,000,000 P2,000,000 P1,000,000
Investment income 125,000 125,000 –
Total 3,125,000 2,125,000 1,000,000
Cost and expense 1,500,000 1,200,000 750,000
Net income P1,625,000 P 925,000 P 250,000

Financial Position:
Assets P3,550,000 P2,850,000 P2,000,000
Investment in Dilawan Company 325,000 325,000 –
Total assets P3,875,000 P3,175,000 P2,000,000
Liabilities P2,100,000 P1,900,000 P1,350,000
Capital stock 1,200,000 P1,000,000 –
Retained earnings 575,000 275,000 –
Ventures, Capital – – 650,000
Total liabilities and capital P3,875,000 P3,175,000 P2,000,000

How much would be the total liabilities to be reported by Sibuyas Company on


December 31, Year 1 is:
P3,250,000 P3,400,000 P2,575,000 P1,900,000
General Feedback
Dilawan Company
Total liabilities 1,900,000.00
*note: since the Dilawan company is a joint venture; the parties shall account
for their investment using the equity method; thus, the liabilities of the JV shall
be reported only by the JV entity.

On January 1, Year 1, two real estate companies, Woodsgate and Deca, set up a
separate vehicle, Royal Pines Company, for the purpose of acquiring and operating a
shopping center. The contractual arrangement between the parties establishes joint
control of the activities that are conducted in Royal Pines Company. The main feature
of Royal Pines’ Legal form is that the entity, not the parties, has rights to the assets,
and obligations for the liabilities, relating to the arrangement. These activiites include
the rental of the retail units, managing the car park, maintaining the center and its
equipment, such as lifts, and building the reputation and customer base for the center
as a whole.
As a result, Woodsgate Company paid P1.6 million for 50,000 shares of Royal Pines’
voting common stock, which represents a 40% investment. No allocation to goodwill
or other specific account was mad the joint control over Royal Pines is achieved by
this acquisition and so Woodsgate applies the equity method. Royal Pines’ distributed
a dividend of P2 per share during the year and reported net income of P560,000. What
is the balance in the Investment in Royal Pines account found in Woodsgate’s
financial records as of December 31, Year 1?
1,844,000 1,884,000 1,724,000 1,784,000
General Feedback
Woodsgate and Deca
Initial Investment 1,600,000.00
Profit share 224,000.00
Dividend share (100,000.00)
Investment, ending 1,724,000.00
Apple Inc. and Samsung Inc. Incorporated an entity named Sample Inc. where in the
parties will have voting rights in the decision affecting the relevant activities of the
arrangement. The contract provides that unanimous consent by the parties is necessary for
the validity of Sample’s corporate act. The purpose of the arrangement is for Sample Inc.
to manufacture parts for the parties own manufacturing processes. The assets and
liabilities held in Sample Inc. are in are name of Sample Inc. what is the classification of
the interest of Apple Inc. and Samsung Inc. in Sample Inc. based on Sample Inc.’s Legal
form only.

It shall be classified as Investment in Subsidiary because of the presence of control.

It shall be classified as Investment in Associate because f the presence of significant


influence.

It shall be classified as Joint Arrangement accounted for as Investment in Joint Venture


under Equity Method because Sample Inc. holds title over the assets of the venture.

It shall be classified as Joint Arrangement accounted for as Joint Operation because in


case of doubt, it shall be resolved in favour of Joint Operation.
General Feedback
It shall be classified as Joint Arrangement accounted for as Investment in Joint Venture
under Equity Method because Sample Inc. holds title over the assets of the venture.

A party to a joint operation sells an asset to the operation. The profit it can realise is:

None

100%

100% - the party’s share, until the asset is sold by the operation

The selling operator’s share


General Feedback
100% - the party’s share, until the asset is sold by the operation
Goodwill arising from a business combination is

amortized over 40 years or its useful life, whichever is longer.

amortized over 40 years or its useful life, whichever is longer.

amortized over 40 years or its useful life, whichever is shorter.

never amortized.
General Feedback
never amortized.

A contingent consideration agreement was made on Jan. 1, Year 1, wherein an additional


cash payment would be made on Jan. 1, Year 3, equal to twice the amount by which
average annual earnings of the Hanes Division exceed P25,000 per year, prior to January
1, Year 3. Net income was P50,000 in Year 1 and P60,000 in Year 2. How much
adjustment will be made to goodwill on January 1, Year 3?

P60,000

P120,000

P85,000

none
General Feedback
none

On January 1, Year 2, Carlito Company acquired 80% interests in Harries Company for
P2,000,000 cash. The stockholder’s equity of Harries at the time of acquisition is
P1,875,000. On January 1, Year 2, NCI is measured at its implied fair value. The excess
of cost over books value of interest acquired is allocated to the following assets:
Inventories P100, 000 (sold in Year 2)

Building P200, 000 (5- year remaining life)

During Year 2, Harries Company reported total comprehensive income of P500,000 and
paid dividend for P100,000.

What is the NCI in net assets of subsidiary on December 31, Year 2?

P455,000

P552,000

P495,000

P795,900
General Feedback
P552,000

How much is the profit attributable to the Controlling Interest?

40,000

32,000

24,000

8,000
General Feedback
Suggested Solution
Pact and Sact
Profit reported by subsidiary 40,000.00
Multiply by number of months with control 9/12
Multiply by percentage of ownership 80%

Profit attributable to Controlling


Interest 24,000.00

*It was assumed that no further equity transactions transpired from


the acquisition date since no data is available.

On January 1, Year 1 SST Company purchased a computer with an expected life of 5 years. On
January 1, Year 3 SST Company sold the computer to PMN corporation and recorded the
following entry:
Cash P39, 000
Accumulated Depreciation 16, 000
Computer Equipment 40, 000
Gain on sale of equipment 15, 000

PMN Corporation holds 60% of the voting shares of SST Company. SST Company and PMN
Corporation reported income from its own operations of P45, 000 and P85, 000 for Year 3
respectively. There is no change in the estimated life of the equipment as a result of
intercompany sale.

How much is the income attributable to the Non-Controlling Interest for Year 3?

12,000

14,000

18,000

21,000
General Feedback
PMN Corporation and SST Company
NCI
Profit - Parent (own operation)
Profit - Subsidiary 18,000
Intercompany sale of PPE
Eliminate Gain (6,000)
Amortization of gain 2,000

Total 14,000

On January 1, Year 1 SST Company purchased a computer with an expected life of 5 years. On
January 1, Year 3 SST Company sold the computer to PMN corporation and recorded the
following entry:
Cash P39, 000
Accumulated Depreciation 16, 000
Computer Equipment 40, 000
Gain on sale of equipment 15, 000

PMN Corporation holds 60% of the voting shares of SST Company. SST Company and PMN
Corporation reported income from its own operations of P45, 000 and P85, 000 for Year 3
respectively. There is no change in the estimated life of the equipment as a result of
intercompany sale.

What is the consolidated total comprehensive income attributable to parent for Year 3?

P103, 000

P106, 000

P112, 000
P130, 000
General Feedback
PMN Corporation and SST Company
EHP NCI Consolidated
Profit - Parent (own operation) 85,000 85,000
Profit - Subsidiary 27,000 18,000 45,000
Intercompany sale of PPE
Eliminate Gain (9,000) (6,000) (15,000)
Amortization of gain 3,000 2,000 5,000

Total 106,000 14,000 120,000

Three joint operators are involved in a joint operation that manufactures ships chandlery.
At the beginning of the year the joint operation held P50,000 in cash. During the year the
joint operation incurred the following expenses: Wages paid P20,000, Overheads accrued
P10,000. Additionally, creditors amounting to P40,000 were paid and the joint operators
contributed P15,000 cash each to the joint operation. The balance of cash held by the
joint operation at the end of the year is:

P5,000

P25,000

P35,000

P75,000
General Feedback
Suggested Solution
Cash, beginning 50,000
Joint operators' additional investment 45,000
Disbursments
Wages (20,000)
Payment of accounts (40,000)

Cash, ending 35,000

All of the following data may be needed to determine the fair value of a forward contract
at any point in time except The forward rate when the forward contract was entered
into. The current forward rate for a contract that matures on the same date as the
forward contract entered into. The future spot rate. A discount rate.
General Feedback
The future spot rate.

Meisner Co. ordered parts costing §100,000 from a foreign supplier on May 12 when the
spot rate was PhP.24 per §. A one-month forward contract was signed on that date to
purchase §100,000 at a forward rate of PhP.25 per §. On June 12, when the parts were
received and payment was made, the spot rate was PhP.28 per §. At what amount should
inventory be reported? PhP0. PhP28,000. PhP24,200. PhP25,000.
General Feedback
PhP28,000.
Which of the following is not a characteristic of joint arrangement classified as joint
operation? The operators have right to the assets and obligation for the liabilities of
the operation The legal form of the separate vehicle does not confer separation
between the operators and the separate vehicle The operators have right to the net
assets of the operation. Without a separate vehicle, the joint arrangement is always
classified as joint operation
General Feedback
The operators have right to the net assets of the operation.

IFRS 11 defines joint arrangement as an arrangement over which two or more parties
have joint control which is the contractually agreed sharing of control of an arrangement,
which exists only when the decisions about the relevant activities require the unanimous
consent of the parties sharing control. What is the classification of the joint arrangement
if the business formed is not structured through a separate vehicle? Joint operation
only Joint venture only Either Joint operation or Joint venture depending on the
substance of the transaction Jointly controlled entity
General Feedback
Joint operation only

Which of the following is not a characteristics of joint arrangement classified as joint


venture? The ventures have right t the net assets of the joint venture. The venturer
accounts for its interest in joint venture by recognizing Investment in Joint Venture and
accounting it using Equity Method under IAS 28 With separate vehicle, the joint
arrangement is always classified as joint venture The venture recognizes its share in
net income /(net loss from joint venture by increase/(decrease) in Investment in Joint
Venture account and records receipt of cash/property dividend from the venture with a
decrease in Investment in Joint Venture account.
General Feedback
With separate vehicle, the joint arrangement is always classified as joint venture

Apple Inc. and Samsung Inc. Incorporated an entity named Sample Inc. where in the
parties will have voting rights in the decision affecting the relevant activities of the
arrangement. The contract provides that unanimous consent by the parties is necessary for
the validity of Sample’s corporate act. The purpose of the arrangement is for Sample Inc.
to manufacture parts for the parties own manufacturing processes. The assets and
liabilities held in Sample Inc. are in are name of Sample Inc. what is the classification of
the interest of Apple Inc. and Samsung Inc. in Sample Inc. based on Sample Inc.’s Legal
form only. It shall be classified as Investment in Subsidiary because of the presence of
control It shall be classified as Investment in Associate because f the presence of
significant influence. It shall be classified as Joint Arrangement accounted for as
Investment in Joint Venture under Equity Method because Sample Inc. holds title over
the assets of the venture. It shall be classified as Joint Arrangement accounted for as
Joint Operation because in case of doubt, it shall be resolved in favour of Joint Operation.
General Feedback
It shall be classified as Joint Arrangement accounted for as Investment in Joint Venture
under Equity Method because Sample Inc. holds title over the assets of the venture.

Which statement is true regarding a foreign currency option? A foreign currency


option gives the holder the obligation to buy or sell foreign currency in the future. A
foreign currency option gives the holder the obligation only sell foreign currency in the
future. A foreign currency option gives the holder the obligation to only buy foreign
currency in the future. A foreign currency option gives the holder the right but not the
obligation to buy or sell foreign currency in the future.
General Feedback
A foreign currency option gives the holder the right but not the obligation to buy or sell
foreign currency in the future.

On December 1, 2018, Keenan Company sold merchandise to Velez Company of


Spain for 150,000 euro. Payment is due on February 1, 2008. Keenan entered into a
forward exchange contract on December 1, 2018, to deliver 150,000 euro on February
1, 2008 for 0.97. Keenan chose to use a foreign currency option to hedge this foreign
currency asset designated as a cash flow hedge. Relevant exchange rates follow:
Spot Rate Option
Premium
December 1, 2018 0.97 0.05
December 31, 2018 0.95 0.04
February 1, 2008 0.94 0.03

Compute the value of the foreign currency option at December 1, 2018.


6,000. 4,500. 3,000. 7,500.
General Feedback
7,500.

Which of the following is false relating to reverse acquisition? Reverse acquisition


may transpire only if the consideration transferred includes shares of stocks of the
acquiring entity. The acquirer in a reverse acquisition is designated as the legal
acquiree. Non-controlling interest shareholders in a reverse acquisition are
shareholders of the accounting acquirer entity Non-controlling interest in a reverse
acquisition may be valued at the fair value of the shares held by the non-controlling
interest shareholders
General Feedback
Non-controlling interest in a reverse acquisition may be valued at the fair value of the
shares held by the non-controlling interest shareholders
Which of the following is true pertaining to reverse acquisition? Legal acquirer is the
accounting acquirer Legal acquirer is the accounting acquiree Non-controlling
interest are shareholders of legal acquirer Non-controlling interest are shareholders of
accounting acquiree
General Feedback
Legal acquirer is the accounting acquiree

ATC corp purchases raw material from its foreign supplier, JLB, on May 8. Payment
of 2,000,000 foreign currency units (FC) is due in 30 days. May 31 is ATC's fiscal
year-end. The pertinent exchange rates were as follows:
Indirect Exchange
rates
May 8 0.8000
May 31 0.7937
June 7 0.8333

How much will it cost, in terms of the Functional Currency, ATC to finally pay the
payable on June 7?
1,666,667. 2,520,000. 2,500,000. 2,400,000.
General Feedback
2,400,000.

Brisco Bricks purchases raw material from its foreign supplier, Bolivian Clay, on May
8. Payment of 2,000,000 foreign currency units (FC) is due in 30 days. May 31 is
Brisco's fiscal year-end. The pertinent exchange rates were as follows:
Selling Spot Buying Spot
Rate Rate
May 8 P 1.25 P 1.23
May 31 P 1.26 P 1.25
June 7 P 1.20 P 1.21

How much Foreign Exchange Gain or Loss should Brisco record on May 31?
20,000 gain. 20,000 loss. 40,000 gain. 40,000 loss.
General Feedback
20,000 loss.

On January 1,2017, the company entered into a two-year P100,000 variable interest rate
loan. In the first year of the loan, the interest rate is 10%. In its second year, the interest
rate is equal to the prime lending rate on January 1,2018. The company does not want to
bear the risk associated with the uncertain interest rate in the second year. Accordingly,
on January 1,2018, the company enters into a pay-fixed, receive-variable interest rate
swap with a speculator. The swap obligates the company to pay the speculator a fixed
amount of P10,000 (100,000 x .10) on December 31,2018. In return, the company will
receive from the speculator on December 31,2018 a variable amount received from the
speculator on December 31,2018 a variable amount equal to P100,000 multiplied by the
prime lending rate on January 1,2018. This amount received from the speculator is
exactly enough to pay the interest due on the variable-rate loan in 2018. Typically,
interest rate swaps such as this are settled with a single net cash payment rather than the
actual payment of P10,000 and receipt of the variable amount.

What net amount will the company pay or receive on December 31,2018 if the prime
lending rate on January 1,2018 is 15%.

P5,000 net payment P10,000 net payment 5,000 net receive 15,000 net
receive
General Feedback
5,000 net receive

Which of the following statements is true concerning hedge accounting? Hedges of


foreign currency firm commitments are used for future sales only. Hedges of foreign
currency firm commitments are used for future purchases only. Hedges of foreign
currency firm commitments are used for current purchases or sales. Hedges of foreign
currency firm commitments are used for future sales or purchases.
General Feedback
Hedges of foreign currency firm commitments are used for future sales or purchases.

A speculative derivative would be similar to which type of hedge? An option


designated as a cash flow hedge. An option designated as a fair value hedge. A
forward contract designated as a cash flow hedge. A forward contract designated as a
fair value hedge.
General Feedback
An option designated as a fair value hedge.

On January 1, Year 1, Dabawenyo Company purchased 80 percent of the outstanding shares of


Minero Company for 600,000 in cash. On that date, Minero had 200,000 of capital stock and
500,000 of retained earnings. All of the assets and liabilities of Minero Company were fairly
valued. Goodwill, if any, is not amortized. The January 1, Year 5, inventory of Minero includes
10,000 of merchandise purchased from Dabawenyo in year Year 4 at 125 percent of cost. The
December 31, Year 5, inventory of Minero includes 16,000 of merchandise purchased from
Dabawenyo at the same markup. Minero’s inventories are on a FIFO basis. For year Year 5,
Minero reported net income of 80,000 and paid dividends of 40,000. Dabawenyo’s income from
its own operations is 400,000; it paid dividends of 250,000. If Dabawenyo uses the cost method
on its books, what will be its consolidated net income to retained earnings compared with its
book income? 32,000 higher 30,800 higher 29,800 higher 1,200 lower
General Feedback
EHP NCI Conso

Income - Parent (Dabawenyo) 400,000.00 400,000.00

Income - Subsidiary (Minero) 64,000.00 16,000.00 80,000.00


Intercompany sale -
Downstream

RGP from Inventory, beg 2,000.00 2,000.00

DGP from Inventory, end (3,200.00) (3,200.00)


Total
462,800.00 16,000.00 478,800.00

30,800 higher

On January 1, Year 1, Polk Corp. and Strass Corp. had condensed balance sheets as follows:
Polk Strass
Current assets $ 70,000 $20,000
Noncurrent assets 90,000 40,000
Total assets $160,000 $60,000
Current liabilities 30,000 10,000
Long-term debt 50,000 --
Stockholders’ equity 80,000 50,000
Total liabilities and stockholders’ equity 160,000 60,000
On January 2, Year 1, Polk borrowed $60,000 and used the proceeds to purchase 90% of the
outstanding common shares of Strass. This debt is payable in ten equal annual
principal payments, plus interest, beginning December 30, Year 1. The excess cost of the
investment over Strass’ book value of acquired net assets should be allocated 60% to inventory
and 40% to goodwill. On January 1, Year 1, the fair
value of Polk shares held by noncontrolling parties was $10,000.

On Polk’s January 2, Year 1 consolidated balance sheet


Current liabilities should be

$50,000 $46,000 $40,000 $30,000


General Feedback
In the consolidated balance sheet, the parent company’s “investment in subsidiary”
account should be eliminated and replaced by the assets and liabilities of the subsidiary.
Therefore, the consolidated balance sheet should include the current liabilities of both
companies, plus the current portion of the debt incurred on 1/2/11 ($60,000 ÷10 =
$6,000). Thus, current liabilities should be reported at $46,000 as computed below.
$46,000

You might also like